Complex Health Challenges Flashcards

NURS 4036 Erin's Midterm - October 30th

1
Q

Name 3 clinical manifestations of chronic heart failure.
(week 1)

A

Fatigue - due to decreased CO, impaired perfusion to vital organs, decreased oxygenation, and anemia

Dyspnea – caused by pulmonary pressures secondary to interstitial and alveolar edema, can occur with mild exertion or at rest, Orthopnea occurs when the patient is in a recumbent position

Paroxysmal nocturnal dyspnea – Due to reabsorption of fluid from dependent areas when the patient lies flat

Cough – dry, nonproductive cough
Tachycardia – due to SNS stimulation
Edema –dependent areas, abdomen, lungs, sacral and scrotal areas

Nocturia – due to impaired renal perfusion during the day, when lying flat blood moves back into the circulatory system

Skin changes – due to increased capillary oxygen extraction – skin appears dusk, lower extremities are shiny, swollen, and diminished hair growth

Behavioural changes – due to decreased cerebral perfusion and/or impaired gas exchange = restlessness, confusion, impaired memory, *Many HF patients have coexisting psychological disorders (depression)

Chest pain – due to decreased coronary artery perfusion
Weight changes – progressive weight gain from fluid retention and renal failure

How well did you know this?
1
Not at all
2
3
4
5
Perfectly
2
Q

Name 2 complications of heart failure.
(week 1)

A

Pleural Effusion – fluid leaking into pleural space

Dysrhythmias – enlargement of heart changes normal electrical pathways (i.e., atrial fibrillation, ventricular tachycardia)

Left Ventricular Thrombus – increased LV and decreased CO increase risk of thrombus formation, emboli can develop from thrombus leading to stroke

Hepatomegaly – venous system backing up into liver leading to impaired liver function, cirrhosis can develop

Renal Failure – decreased CO leads to hypoperfusion of kidneys = renal insufficiency or failure

How well did you know this?
1
Not at all
2
3
4
5
Perfectly
3
Q

How do you diagnose heart failure?
(week 1)

A

Echocardiography – distinguish HFrEF from HFpEF

Measurement of BNP or N-terminal-pro-BNP – help distinguish dyspnea caused by HF from other causes of dyspnea

Chest Xray – can help distinguish pleural effusions from other causes of dyspnea (i.e., pneumonia), can also visualize cardiomegaly

How well did you know this?
1
Not at all
2
3
4
5
Perfectly
4
Q

What are the two medication conditions are primary risk factors for heart failure?
(week 1)

A

Coronary artery disease (CAD) and hypertension (HTN) are the primary risk factors for HF

Diabetes, smoking, obesity, and high serum cholesterol are other risk factors

How well did you know this?
1
Not at all
2
3
4
5
Perfectly
5
Q

What are the two categories of heart failure?
(week 1)

A

Two categories of HF: reduced ejection fraction (HF-REF) and preserved ejection fraction (HF-PEF)

How well did you know this?
1
Not at all
2
3
4
5
Perfectly
6
Q

T/F: Reduced ejection fraction (HF-REF) is the most common form of heart failure.
(week 1)

A

True

More on HF-REF:
Heart can’t pump blood effectively – impaired contractile function, increased afterload (hypertension), cardiomyopathy and mechanical abnormalities. Left ventricle loses ability to generate enough pressure to eject blood forward through aorta.

How well did you know this?
1
Not at all
2
3
4
5
Perfectly
7
Q

Describe the patho of reserved ejection fraction (HF-PEF).
(week 1)

A

Aka diastolic HF – inability of ventricles to relax and fill during diastole

Decreased filling results in decreased stroke volume and cardiac output (CO). Results in venous engorgement in both pulmonary and vascular systems.

Most often results from left ventricular hypertrophy (due to hypertension), myocardial ischemia, valve disease, or cardiomyopathy

How well did you know this?
1
Not at all
2
3
4
5
Perfectly
8
Q

Define 2 nursing management interventions for acute decompensated heart failure.
(week 1)

A

Decrease intravascular volume > loop diuretic med admin

Decrease Venous return > Elevating the head of the bed with feet dangling

Decrease Afterload > Monitor vital signs - decreased systemic vascular resistance (lower BP)

Improving Gas Exchange > Administer oxygen if sats <90%

Improving Cardiac Function >Aggressive complex therapies including inotropic therapies and hemodynamic monitoring

Reduce Anxiety > Calm approach, morphine

How well did you know this?
1
Not at all
2
3
4
5
Perfectly
9
Q

Name the two kinds of diuretics used to treat chronic heart failure and describe their function.
(week 1)

A

Diuretics
- mobilize edematous fluid, reduce pulmonary venous pressure, reduce preload, improve cardiac function

  1. Loop diuretics (i.e., furosemide [Lasix]) –
    Act on the ascending loop of Henle to promote excretion of sodium, chloride, and water used in acute HF and pulmonary edema, risk for hypokalemia
  2. Thiazide diuretics (i.e., hydrochlorothiazide) –
    Inhibit sodium reabsorption from distal tubule promoting excretion of sodium and water
    Treat edema and HTN
How well did you know this?
1
Not at all
2
3
4
5
Perfectly
10
Q

The nurse is preparing to administer digoxin to a patient with heart failure. In preparation, laboratory results are reviewed with the following findings: sodium 139 mEq/L, potassium 5.6 mEq/L, chloride 103 mEq/L, and glucose 106 mg/dL. What should the nurse do next?

A. Withhold the daily dose until the following day.
B. Withhold the dose and report the potassium level.
C. Give the digoxin with a salty snack, such as crackers.
D. Give the digoxin with extra fluids to dilute the sodium level.

(week 1)

A

B. Withhold the dose and report the potassium level.

The normal potassium level is 3.5 to 5.0 mEq/L. The patient is hyperkalemic, which makes the patient more prone to digoxin toxicity. For this reason, the nurse should withhold the dose and report the potassium level. The physician may order the digoxin to be given once the potassium level has been treated and decreases to within normal range.

How well did you know this?
1
Not at all
2
3
4
5
Perfectly
11
Q

What is the priority assessment by the nurse caring for a patient receiving IV nesiritide (Natrecor) to treat heart failure?

A. Urine output
B. Lung sounds
C. Blood pressure
D. Respiratory rate
(week 1)

A

C. Blood pressure

Although all identified assessments are appropriate for a patient receiving IV nesiritide, the priority assessment would be monitoring for hypotension, the main adverse effect of nesiritide.

How well did you know this?
1
Not at all
2
3
4
5
Perfectly
12
Q

A patient admitted with heart failure appears very anxious and complains of shortness of breath. Which nursing actions would be appropriate to alleviate this patient’s anxiety (select all that apply)?

A. Administer ordered morphine sulfate.
B. Position patient in a semi-Fowler’s position.
C. Position patient on left side with head of bed flat.
D. Instruct patient on the use of relaxation techniques.
E. Use a calm, reassuring approach while talking to patient.
(week 1)

A

A, B, D, E.

Morphine sulfate reduces anxiety and may assist in reducing dyspnea. The patient should be positioned in semi-Fowler’s position to improve ventilation that will reduce anxiety. Relaxation techniques and a calm reassuring approach will also serve to reduce anxiety.

How well did you know this?
1
Not at all
2
3
4
5
Perfectly
13
Q

A male patient with a long-standing history of heart failure has recently qualified for hospice care. What measure should the nurse now prioritize when providing care for this patient?

A. Taper the patient off his current medications.
B. Continue education for the patient and his family.
C. Pursue experimental therapies or surgical options.
D. Choose interventions to promote comfort and prevent suffering.
(week 1)

A

D. Choose interventions to promote comfort and prevent suffering.

The central focus of hospice care is the promotion of comfort and the prevention of suffering. Patient education should continue, but providing comfort is paramount. Medications should be continued unless they are not tolerated. Experimental therapies and surgeries are not commonly used in the care of hospice patients.

How well did you know this?
1
Not at all
2
3
4
5
Perfectly
14
Q

What should the nurse recognize as an indication for the use of dopamine (Intropin) in the care of a patient with heart failure?

A. Acute anxiety
B. Hypotension and tachycardia
C. Peripheral edema and weight gain
D. Paroxysmal nocturnal dyspnea (PND)
(week 1)

A

B. Hypotension and tachycardia

Dopamine is a β-adrenergic agonist whose inotropic action is used for treatment of severe heart failure accompanied by hemodynamic instability. Such a state may be indicated by tachycardia accompanied by hypotension. PND, anxiety, edema, and weight gain are common signs and symptoms of heart failure, but these do not necessarily warrant the use of dopamine.

How well did you know this?
1
Not at all
2
3
4
5
Perfectly
15
Q

A patient with a recent diagnosis of heart failure has been prescribed furosemide (Lasix) in an effort to physiologically do what for the patient?

A. Reduce preload.
B. Decrease afterload.
C. Increase contractility.
D. Promote vasodilation.
(week 1)

A

A. Reduce preload.

Diuretics such as furosemide are used in the treatment of HF to mobilize edematous fluid, reduce pulmonary venous pressure, and reduce preload. They do not directly influence afterload, contractility, or vessel tone.

How well did you know this?
1
Not at all
2
3
4
5
Perfectly
16
Q

A patient with a diagnosis of heart failure has been started on a nitroglycerin patch by his primary care provider. What should this patient be taught to avoid?

A. High-potassium foods
B. Drugs to treat erectile dysfunction
C. Nonsteroidal antiinflammatory drugs
D. Over-the-counter H2-receptor blockers
(week 1)

A

B. Drugs to treat erectile dysfunction

The use of erectile drugs concurrent with nitrates creates a risk of severe hypotension and possibly death. High-potassium foods, NSAIDs, and H2-receptor blockers do not pose a risk in combination with nitrates.

How well did you know this?
1
Not at all
2
3
4
5
Perfectly
17
Q

A stable patient with acute decompensated heart failure (ADHF) suddenly becomes dyspneic. Before positioning the patient on the bedside, what should the nurse assess first?

A. Urine output
B. Heart rhythm
C. Breath sounds
D. Blood pressure
(week 1)

A

D. Blood pressure

The nurse should evaluate the blood pressure before dangling the patient on the bedside because the blood pressure can decrease as blood pools in the periphery and preload decreases. If the patient’s blood pressure is low or marginal, the nurse should put the patient in the semi-Fowler’s position and use other measures to improve gas exchange.

How well did you know this?
1
Not at all
2
3
4
5
Perfectly
18
Q

After having an MI, the nurse notes the patient has jugular venous distention, gained weight, developed peripheral edema, and has a heart rate of 108/minute. What should the nurse suspect is happening?

A. ADHF
B. Chronic HF
C. Left-sided HF
D. Right-sided HF
(week 1)

A

D. Right-sided HF

An MI is a primary cause of heart failure. The jugular venous distention, weight gain, peripheral edema, and increased heart rate are manifestations of right-sided heart failure.

How well did you know this?
1
Not at all
2
3
4
5
Perfectly
19
Q

The patient with chronic heart failure is being discharged from the hospital. What information should the nurse emphasize in the patient’s discharge teaching to prevent progression of the disease to ADHF?

A. Take medications as prescribed.
B. Use oxygen when feeling short of breath.
C. Only ask the physician’s office questions.
D. Encourage most activity in the morning when rested.
(week 1)

A

A. Take medications as prescribed.

The goal for the patient with chronic HF is to avoid exacerbations and hospitalization. Taking the medications as prescribed along with nondrug therapies such as alternating activity with rest will help the patient meet this goal. If the patient needs to use oxygen at home, it will probably be used all the time or with activity to prevent respiratory acidosis.

How well did you know this?
1
Not at all
2
3
4
5
Perfectly
20
Q

The home care nurse visits a 73-year-old Hispanic woman with chronic heart failure. Which clinical manifestations, if assessed by the nurse, would indicate acute decompensated heart failure (pulmonary edema)?

A. Fatigue, orthopnea, and dependent edema
B. Severe dyspnea and blood-streaked, frothy sputum
C. Temperature is 100.4o F and pulse is 102 beats/minute
D. Respirations 26 breaths/minute despite oxygen by nasal cannula
(week 1)

A

B. Severe dyspnea and blood-streaked, frothy sputum

Clinical manifestations of pulmonary edema include anxiety, pallor, cyanosis, clammy and cold skin, severe dyspnea, use of accessory muscles of respiration, a respiratory rate > 30 breaths per minute, orthopnea, wheezing, and coughing with the production of frothy, blood-tinged sputum. Auscultation of the lungs may reveal crackles, wheezes, and rhonchi throughout the lungs. The heart rate is rapid, and blood pressure may be elevated or decreased.

How well did you know this?
1
Not at all
2
3
4
5
Perfectly
21
Q

A 54-year-old male patient who had bladder surgery 2 days ago develops acute decompensated heart failure (ADHF) with severe dyspnea. Which action by the nurse would be indicated first?

A. Perform a bladder scan to assess for urinary retention.
B. Restrict the patient’s oral fluid intake to 500 mL per day.
C. Assist the patient to a sitting position with arms on the overbed table.
D. Instruct the patient to use pursed-lip breathing until the dyspnea subsides.
(week 1)

A

C. Assist the patient to a sitting position with arms on the overbed table.

The nurse should place the patient with ADHF in a high Fowler’s position with the feet horizontal in the bed or dangling at the bedside. This position helps decrease venous return because of the pooling of blood in the extremities. This position also increases the thoracic capacity, allowing for improved ventilation. Pursed-lip breathing helps with obstructive air trapping but not with acute pulmonary edema. Restricting fluids takes considerable time to have an effect.

How well did you know this?
1
Not at all
2
3
4
5
Perfectly
22
Q

A 70-year-old woman with chronic heart failure and atrial fibrillation asks the nurse why warfarin (Coumadin) has been prescribed for her to continue at home. Which response by the nurse is accurate?

A. “The medication prevents blood clots from forming in your heart.”
B. “The medication dissolves clots that develop in your coronary arteries.”
C. “The medication reduces clotting by decreasing serum potassium levels.”
D. “The medication increases your heart rate so that clots do not form in your heart.”
(week 1)

A

A. “The medication prevents blood clots from forming in your heart.”

Chronic heart failure causes enlargement of the chambers of the heart and an altered electrical pathway, especially in the atria. When numerous sites in the atria fire spontaneously and rapidly, atrial fibrillation occurs. Atrial fibrillation promotes thrombus formation within the atria with an increased risk of stroke and requires treatment with cardioversion, antidysrhythmics, and/or anticoagulants. Warfarin is an anticoagulant that interferes with hepatic synthesis of vitamin K-dependent clotting factors.

How well did you know this?
1
Not at all
2
3
4
5
Perfectly
23
Q

While assessing an older adult patient, the nurse notes jugular venous distention (JVD) with the head of the patient’s bed elevated 45 degrees. What does this finding indicate?

a. Decreased fluid volume
b. Jugular vein atherosclerosis
c. Increased right atrial pressure
d. Incompetent jugular vein valves
(week 1)

A

C - Increased right atrial pressure

The jugular veins empty into the superior vena cava and then into the right atrium, so JVD with the patient sitting at a 45-degree angle reflects increased right atrial pressure. JVD is an indicator of excessive fluid volume (increased preload), not decreased fluid volume. JVD is not caused by incompetent jugular vein valves or atherosclerosis.

How well did you know this?
1
Not at all
2
3
4
5
Perfectly
24
Q

The nurse is caring for a patient who is receiving IV furosemide (Lasix) and morphine for the treatment of acute decompensated heart failure (ADHF) with severe orthopnea. Which clinical finding is the best indicator that the treatment has been effective?

a. Weight loss of 2 lb in 24 hours
b. Hourly urine output greater than 60 mL
c. Reduced dyspnea with the head of bed at 30 degrees
d. Patient denies experiencing chest pain or chest pressure
(week 1)

A

C - Reduced dyspnea with the head of bed at 30 degrees

Because the patient’s major clinical manifestation of ADHF is orthopnea (caused by the presence of fluid in the alveoli), the best indicator that the medications are effective is a decrease in dyspnea with the head of the bed at 30 degrees. The other assessment data may also indicate that diuresis or improvement in cardiac output has occurred but are not as specific to evaluating this patient’s response.

How well did you know this?
1
Not at all
2
3
4
5
Perfectly
25
Q

Describe a non-pharm therapy/intervention used to treat heart failure.
(week 1)

A

Oxygen
- helps relieve dyspnea and fatigue, pulse oximetry monitoring

Self-management teaching
- teaching to pt and families about recognizing early signs and symptoms of decompensation, supporting patients to manage their illness

How well did you know this?
1
Not at all
2
3
4
5
Perfectly
26
Q

What is the first line drug therapy for heart failure?
(week 1)

A

Angiotensin Converting Enzyme (ACE) Inhibitors
- first-line therapy in HF (useful in both systolic and diastolic HF) (i.e., ramipril [Atlace], perindopril [Coversyl])
- Block conversion of angiotensin I to angiotensin II, as a result, aldosterone levels are also reduced
Reduces systemic vascular resistance, maintains tissue perfusion, ventricular filling pressure
- Risk for hypotension, chronic cough, and renal insufficiency in high-does
Risk for life-threatening angioedema (rare)

How well did you know this?
1
Not at all
2
3
4
5
Perfectly
27
Q

What role does Beta-Blockers have in treating chronic heart failure?
(week 1)

A

B-Adrenergic Blockers
- block negative effects of SNS on the failing heart including increased HR (i.e., metoprolol, bisoprolol)
- Risk for reduced myocardial contractility
- Can also cause bradycardia, fatigue, asthma exacerbations, edema

How well did you know this?
1
Not at all
2
3
4
5
Perfectly
28
Q

Identify 1 nursing diagnosis for a chronic heart failure patient.
(week 1)

A

Impaired Gas Exchange
Cardiac Care
Hypervolemia Management
Activity Intolerance

How well did you know this?
1
Not at all
2
3
4
5
Perfectly
29
Q

T/F: Left sided heart failure is most common.
(week 1)

A

True

How well did you know this?
1
Not at all
2
3
4
5
Perfectly
30
Q

What is the reason for ordering the beta blocker carvedilol in a heart failure patient?

a. To increase urine output
b. To cause peripheral vasodilation.
c. To increase the contractility of the heart
d. reduce cardiac stimulation from catecholamines.
(week 1)

A

ANS: D
Beta-blockers reduce or prevent stimulation of the heart from circulating catecholamines.
A – Diuretics increase urine output
B – ACE inhibitors cause peripheral vasodilation
C – Inotropic medications (i.e. digoxin) increase the contractility of the heart

How well did you know this?
1
Not at all
2
3
4
5
Perfectly
31
Q

What is the action of the digoxin? Digoxin?
a. causes systemic vasodilation
b. promotes the excretion of sodium and water in the renal tubules
c. increases cardiac contractility and cardiac output
d. blocks sympathetic nervous system stimulation to the heart.
(week 1)

A

ANS: C
Digoxin works by increasing cardiac contractility and cardiac output.

Digoxin works by inducing an increase in intracellular sodium that will drive an influx of calcium into the heart and cause an increase in contractility – cardiac output increases with a subsequent decrease in filling pressures
Digoxin also slows the heart rate

How well did you know this?
1
Not at all
2
3
4
5
Perfectly
32
Q

Which findings from your patients assessment would indicate an increased possibility of digoxin toxicity? Explain your answer.
a. Serum potassium level of 2.2 mEq/L
b. Serum sodium level of 139 mEq/L
c. Apical heart rate of 64 beats/minute
d. Digoxin level 1.6 ng/mL
(week 1)

A

ANS: A
Low potassium levels can increase the potential for digoxin toxicity.
Potassium levels should be monitored carefully during digoxin therapy. The other findings are within normal limits.

Normal serum potassium is 3.5-5 mmol/L
The therapeutic range for digoxin is 0.5-2 ng/dl (0.6- 1.3 nmol/L)
Normal serum sodium is 135-145 mmol/L
Digoxin decreases HR – a HR <60 would be concerning

How well did you know this?
1
Not at all
2
3
4
5
Perfectly
33
Q

Fill in the blanks.
Normal serum potassium is __-__ mmol/L

The therapeutic range for digoxin is __-__ ng/dl

Normal serum sodium is __-__ mmol/L

Digoxin decreases HR. A heart rate of <__ would be concerning
(week 1)

A

Normal serum potassium is 3.5-5 mmol/L

The therapeutic range for digoxin is 0.5-2 ng/dl (0.6- 1.3 nmol/L)

Normal serum sodium is 135-145 mmol/L

Digoxin decreases HR – a HR <60 would be concerning

How well did you know this?
1
Not at all
2
3
4
5
Perfectly
34
Q

When discharging a heart failure patient, we can use the MAWDS mnemonic to highlight key management concepts. Describe the MAWDS concepts.
(week 1)

A

Self Management - MAWDS for Heart Failure

Medications: Take your medications as prescribed by your doctor.
Activity: Stay active every day.
Weight: Weigh yourself each day. Diet: Follow your diet.
Symptoms: Report your symptoms.

How well did you know this?
1
Not at all
2
3
4
5
Perfectly
35
Q

After the teaching session, which statement by the patient indicates a need for further education?
a. “I will weigh myself daily and tell the doctor at my next visit if I am gaining weight.”
b. “I will not add salt when I am cooking.”
c. “I will try to take a short walk around the block with my husband three times a week.”
d. “I will use a pill calendar box to remind me to take my medicine.”

A

ANS: A
If she notices a weight gain, she needs to notify her physician right away, not at the next office visit. This is what brought her back to the hospital in the first place!

How well did you know this?
1
Not at all
2
3
4
5
Perfectly
36
Q

While assessing a 68-yr-old with ascites, the nurse also notes jugular venous distention (JVD) with the head of the patient’s bed elevated 45 degrees. The nurse knows this finding indicates:
a. decreased fluid volume.
b. jugular vein atherosclerosis.
c. increased right atrial pressure.
d. incompetent jugular vein valves.

A

ANS: C
The jugular veins empty into the superior vena cava and then into the right atrium, so JVD with the patient sitting at a 45-degree angle reflects increased right atrial pressure. JVD is an indicator of excessive fluid volume (increased preload), not decreased fluid volume. JVD is not caused by incompetent jugular vein valves or atherosclerosis

How well did you know this?
1
Not at all
2
3
4
5
Perfectly
37
Q

The nurse is caring for a patient who is receiving IV furosemide (Lasix) and morphine for the treatment of acute decompensated heart failure (ADHF) with severe orthopnea. Which clinicalfinding is the best indicator that the treatment has been effective?
a. Weight loss of 2 lb in 24 hours
b. Hourly urine output greater than 60 mL
c. Reduction in patient complaints of chest pain
d. Reduced dyspnea with the head of bed at 30 degrees

A

ANS: D
Because the patient’s major clinical manifestation of ADHF is orthopnea (caused by the presence of fluid in the alveoli), the best indicator that the medications are effective is a decrease in dyspnea with the head of the bed at 30 degrees. The other assessment data may also indicate that diuresis or improvement in cardiac output has occurred but are not as specific to evaluating this patient’s response.

How well did you know this?
1
Not at all
2
3
4
5
Perfectly
38
Q

Which topic will the nurse plan to include in discharge teaching for a patient with heart failurewith reduced ejection fraction (HFrEF)?
a. Need to begin an aerobic exercise program several times weekly
b. Use of salt substitutes to replace table salt when cooking and at the table
c. Importance of making an annual appointment with the health care provider
d. Benefits and side effects of angiotensin-converting enzyme (ACE) inhibitors

A

ANS: D
The core measures for the treatment of heart failure established by The Joint Commission indicate that patients with an ejection fraction below 40% should receive an ACE inhibitor to decrease the progression of heart failure. Aerobic exercise may not be appropriate for a patientwith this level of heart failure, salt substitutes are not usually recommended because of the risk of hyperkalemia, and the patient will need to see the primary care provider more frequently than annually.

How well did you know this?
1
Not at all
2
3
4
5
Perfectly
39
Q

A patient who has chronic heart failure tells the nurse, “I was fine when I went to bed, but I woke up in the middle of the night feeling like I was suffocating!” The nurse will document this assessment finding as
a. orthopnea.
b. pulsus alternans.
c. paroxysmal nocturnal dyspnea.
d. acute bilateral pleural effusion.

A

ANS: C
Paroxysmal nocturnal dyspnea is caused by the reabsorption of fluid from dependent body areas when the patient is sleeping and is characterized by waking up suddenly with the feelingof suffocation. Pulsus alternans is the alteration of strong and weak peripheral pulses during palpation. Orthopnea indicates that the patient is unable to lie flat because of dyspnea. Pleural effusions develop over a longer time period

How well did you know this?
1
Not at all
2
3
4
5
Perfectly
40
Q

During a visit to a 78-yr-old patient with chronic heart failure, the home care nurse finds that the patient has ankle edema, a 2-kg weight gain over the past 2 days, and complains of “feeling too tired to get out of bed.” Based on these data, a correct nursing diagnosis for the patient is
a. activity intolerance related to fatigue.
b. impaired skin integrity related to edema.
c. disturbed body image related to weight gain.
d. impaired gas exchange related to dyspnea on exertion.

A

ANS: A

How well did you know this?
1
Not at all
2
3
4
5
Perfectly
41
Q

While admitting an 82-yr-old patient with acute decompensated heart failure to the hospital, the nurse learns that the patient lives alone and sometimes confuses the “water pill” with the “heart pill.” When planning for the patient’s discharge the nurse will facilitate a
a. plan for around-the-clock care.
b. consultation with a psychologist.
c. transfer to a long-term care facility.
d. referral to a home health care agency

A

ANS: D
The data about the patient suggest that assistance in developing a system for taking medications correctly at home is needed. A home health nurse will assess the patient’s home situation and help the patient develop a method for taking the two medications as directed. There is no evidence that the patient requires services such as a psychologist consult, long-term care, or around-the-clock home care

How well did you know this?
1
Not at all
2
3
4
5
Perfectly
42
Q

Following an acute myocardial infarction, a previously healthy 63-yr-old develops clinical manifestations of heart failure. The nurse anticipates discharge teaching will include information about
a. b-Adrenergic blockers.
b. calcium channel blockers.
c. digitalis and potassium therapy regimens.
d. angiotensin-converting enzyme (ACE) inhibitors.

A

ANS: D
ACE inhibitor therapy is currently recommended to prevent the development of heart failure in patients who have had a myocardial infarction and as a first-line therapy for patients with chronic heart failure. Digoxin therapy for heart failure is no longer considered a first-line measure, and digoxin is added to the treatment protocol when therapy with other drugs such as ACE-inhibitors, diuretics, and b-adrenergic blockers is insufficient. Calcium channel blockers are not generally used in the treatment of heart failure. The b-adrenergic blockers arenot used as initial therapy for new onset heart failure

How well did you know this?
1
Not at all
2
3
4
5
Perfectly
43
Q

Which diagnostic test will be most useful to the nurse in determining whether a patient admitted with acute shortness of breath has heart failure?
a. Serum troponin
b. Arterial blood gases
c. B-type natriuretic peptide
d. 12-lead electrocardiogram

A

ANS: C
B-type natriuretic peptide (BNP) is secreted when ventricular pressures increase, as they do with heart failure. Elevated BNP indicates a probable or very probable diagnosis of heart failure. A 12-lead electrocardiogram, arterial blood gases, and troponin may also be used in determining the causes or effects of heart failure but are not as clearly diagnostic of heart failure as BNP.

How well did you know this?
1
Not at all
2
3
4
5
Perfectly
44
Q

A patient in the intensive care unit with acute decompensated heart failure (ADHF) complains of severe dyspnea and is anxious, tachypneic, and tachycardic. Several drugs have been ordered for the patient. The nurse’s priority action will be to
a. give PRN IV morphine sulfate 4 mg.
b. give PRN IV diazepam (Valium) 2.5 mg.
c. increase nitroglycerin infusion by 5 mcg/min.
d. increase dopamine infusion by 2 mcg/kg/min

A

ANS: AMorphine improves alveolar gas exchange, improves cardiac output by reducing ventricular preload and afterload, decreases anxiety, and assists in reducing the subjective feeling of dyspnea. Diazepam may decrease patient anxiety, but it will not improve the cardiac output or gas exchange. Increasing the dopamine may improve cardiac output, but it will also increase the heart rate and myocardial oxygen consumption. Nitroglycerin will improve cardiac output and may be appropriate for this patient, but it will not directly reduce anxiety and will not act as quickly as morphine to decrease dyspnea.

How well did you know this?
1
Not at all
2
3
4
5
Perfectly
45
Q

After receiving change-of-shift report on four patients admitted to a heart failure unit, which patient should the nurse assess first?
a. A patient who reported dizziness after receiving the first dose of captopril
b. A patient who is cool and clammy, with new-onset confusion and restlessness
c. A patient who has crackles bilaterally in the lung bases and is receiving oxygen.
d. A patient who is receiving IV nesiritide (Natrecor) and has a blood pressure of 100/62

A

ANS: B
The patient who has “wet-cold” clinical manifestations of heart failure is perfusing inadequately and needs rapid assessment and changes in management. The other patients also should be assessed as quickly as possible but do not have indications of severe decreases in tissue perfusion

How well did you know this?
1
Not at all
2
3
4
5
Perfectly
46
Q

Which assessment finding in a patient admitted with acute decompensated heart failure (ADHF) requires the most immediate action by the nurse?
a. O2 saturation of 88%
b. Weight gain of 1 kg (2.2 lb)
c. Heart rate of 106 beats/min
d. Urine output of 50 mL over 2 hours

A

ANS: A
A decrease in O2 saturation to less than 92% indicates hypoxemia, and the nurse should start supplemental O2 immediately. An increase in apical pulse rate, 1-kg weight gain, and decreases in urine output also indicate worsening heart failure and require nursing actions, butthe low O2 saturation rate requires the most immediate nursing action

How well did you know this?
1
Not at all
2
3
4
5
Perfectly
47
Q

A patient has recently started on digoxin (Lanoxin) in addition to furosemide (Lasix) and captopril for the management of heart failure. Which assessment finding by the home health nurse is a priority to communicate to the health care provider?
a. Presence of 1+ to 2+ edema in the feet and ankles
b. Palpable liver edge 2 cm below the ribs on the right side
c. Serum potassium level 3.0 mEq/L after 1 week of therapy
d. Weight increase from 120 pounds to 122 pounds over 3 days

A

ANS: C
Hypokalemia can predispose the patient to life-threatening dysrhythmias (e.g., premature ventricular contractions) and potentiate the actions of digoxin. Hypokalemia also increases therisk for digoxin toxicity, which can also cause life-threatening dysrhythmias. The other data indicate that the patient’s heart failure requires more effective therapies, but they do not require nursing action as rapidly as the low serum potassium level.

How well did you know this?
1
Not at all
2
3
4
5
Perfectly
48
Q

An outpatient who has chronic heart failure returns to the clinic after 2 weeks of therapy with metoprolol (Toprol XL). Which assessment finding is most important for the nurse to report to the health care provider?
a. 2+ bilateral pedal edema
b. Heart rate of 56 beats/min
c. Complaints of increased fatigue
d. Blood pressure (BP) of 88/42 mm Hg

A

ANS: D
The patient’s BP indicates that the dose of metoprolol may need to be decreased because of hypotension. Bradycardia is a frequent adverse effect of b-adrenergic blockade, but the rate of56 is not unusual though it may need to be monitored. b-Adrenergic blockade initially will worsen symptoms of heart failure in many patients and patients should be taught that some increase in symptoms, such as fatigue and edema, is expected during the initiation of therapy with this class of drugs.

How well did you know this?
1
Not at all
2
3
4
5
Perfectly
49
Q

A 53-yr-old patient with stage D heart failure and type 2 diabetes asks the nurse whether heart transplant is a possible therapy. Which response by the nurse is most accurate?
a. “Your heart failure has not reached the end stage yet.”
b. “You could not manage the multiple complications of that surgery.”
c. “The suitability of a heart transplant for you depends on many factors.”
d. “Because you have diabetes, you would not be a heart transplant candidate.”

A

ANS: C
Indications for a heart transplant include end-stage heart failure (stage D), but other factors such as coping skills, family support, and patient motivation to follow the rigorous posttransplant regimen are also considered. Patients with diabetes who have well-controlled blood glucose levels may be candidates for heart transplant. Although heart transplants can be associated with many complications, there are no data to suggest that the patient could not manage the care

How well did you know this?
1
Not at all
2
3
4
5
Perfectly
50
Q

When developing a teaching plan for a 61-yr-old patient with multiple risk factors for coronary artery disease (CAD), the nurse should focus primarily on the
a. family history of coronary artery disease.
b. elevated low-density lipoprotein (LDL) level.
c. increased risk associated with the patient’s gender.
d. increased risk of cardiovascular disease as people age.

A

ANS: B
Because family history, gender, and age are nonmodifiable risk factors, the nurse should focuson the patient’s LDL level. Decreases in LDL will help reduce the patient’s risk for developingCAD

How well did you know this?
1
Not at all
2
3
4
5
Perfectly
51
Q

Which nursing intervention is likely to be most effective when assisting the patient with coronary artery disease to make appropriate dietary changes?
a. Inform the patient about a diet containing no saturated fat and minimal salt.
b. Help the patient modify favorite high-fat recipes by using monounsaturated oils.
c. Emphasize the increased risk for heart problems unless the patient makes the dietary changes.
d. Give the patient a list of low-sodium, low-cholesterol foods that should be included in the diet.

A

ANS: B
Lifestyle changes are more likely to be successful when consideration is given to the patient’s values and preferences. The highest percentage of calories from fat should come from monounsaturated or polyunsaturated fats. Although low-sodium and low-cholesterol foods areappropriate, providing the patient with a list alone is not likely to be successful in making dietary changes. Completely removing saturated fat from the diet is not a realistic expectation.Up to 7% of calories in the therapeutic lifestyle changes diet can come from saturated fat. Telling the patient about the increased risk without assisting further with strategies for dietary change is unlikely to be successful.

How well did you know this?
1
Not at all
2
3
4
5
Perfectly
52
Q

The nurse is admitting a patient who has chest pain. Which assessment data suggest that the pain is caused by an acute myocardial infarction (AMI)?
a. The pain increases with deep breathing.
b. The pain has lasted longer than 30 minutes.
c. The pain is relieved after the patient takes nitroglycerin.
d. The pain is reproducible when the patient raises the arms.

A

ANS: B
Chest pain that lasts for 20 minutes or more is characteristic of AMI. Changes in pain that occur with raising the arms or with deep breathing are more typical of musculoskeletal pain orpericarditis. Stable angina is usually relieved when the patient takes nitroglycerin.

How well did you know this?
1
Not at all
2
3
4
5
Perfectly
53
Q

Which information from a patient helps the nurse confirm the previous diagnosis of chronic stable angina?
a. “The pain wakes me up at night.”
b. “The pain is level 3 to 5 (0 to 10 scale).”
c. “The pain has gotten worse over the last week.”
d. “The pain goes away after a nitroglycerin tablet.”

A

ANS: D
Chronic stable angina is typically relieved by rest or nitroglycerin administration. The level ofpain is not a consistent indicator of the type of angina. Pain occurring at rest or with increased frequency is typical of unstable angina.

How well did you know this?
1
Not at all
2
3
4
5
Perfectly
54
Q

Which statement made by a patient with coronary artery disease after the nurse has completed teaching about the therapeutic lifestyle changes (TLC) diet indicates that further teaching is needed?
a. “I will switch from whole milk to 1% milk.”
b. “I like salmon and I will plan to eat it more often.”
c. “I can have a glass of wine with dinner if I want one.”
d. “I will miss being able to eat peanut butter sandwiches.”

A

ANS: D

How well did you know this?
1
Not at all
2
3
4
5
Perfectly
55
Q

The nurse suspects that the patient with stable angina is experiencing a side effect of the prescribed drug metoprolol (Lopressor) if the
a. patient is restless and agitated.
b. blood pressure is 90/54 mm Hg.
c. patient complains about feeling anxious.
d. heart monitor shows normal sinus rhythm.

A

ANS: B
Patients taking b-adrenergic blockers should be monitored for hypotension and bradycardia. Because this class of medication inhibits the sympathetic nervous system, restlessness, agitation, hypertension, and anxiety will not be side effects. Normal sinus rhythm is a normal and expected heart rhythm

How well did you know this?
1
Not at all
2
3
4
5
Perfectly
56
Q

Nadolol (Corgard) is prescribed for a patient with chronic stable angina and left ventricular dysfunction. To determine whether the drug is effective, the nurse will monitor for
a. decreased blood pressure and heart rate.
b. fewer complaints of having cold hands and feet.
c. improvement in the strength of the distal pulses.
d. participation in daily activities without chest pain.

A

ANS: D
Because the drug is ordered to improve the patient’s angina, effectiveness is indicated if the patient is able to accomplish daily activities without chest pain. Blood pressure and heart rate may decrease, but these data do not indicate that the goal of decreased angina has been met. The noncardioselective b-adrenergic blockers can cause peripheral vasoconstriction, so the nurse would not expect an improvement in distal pulse quality or skin temperature.

How well did you know this?
1
Not at all
2
3
4
5
Perfectly
57
Q

Heparin is ordered for a patient with a non–ST-segment-elevation myocardial infarction (NSTEMI). What is the purpose of the heparin?
a. Heparin enhances platelet aggregation at the plaque site.
b. Heparin decreases the size of the coronary artery plaque.
c. Heparin prevents the development of new clots in the coronary arteries.
d. Heparin dissolves clots that are blocking blood flow in the coronary arteries.

A

ANS: C

How well did you know this?
1
Not at all
2
3
4
5
Perfectly
58
Q

After an acute myocardial infarction (AMI), a patient ambulates in the hospital hallway. Whenthe nurse evaluates the patient’s response to the activity, which data would indicate that the exercise level should be decreased?
a. O2 saturation drops from 99% to 95%.
b. Heart rate increases from 66 to 98 beats/min.
c. Respiratory rate goes from 14 to 20 breaths/min.
d. Blood pressure (BP) changes from 118/60 to 126/68 mm Hg.

A

ANS: B
A change in heart rate of more than 20 beats over the resting heart rate indicates that the patient should stop and rest. The increases in BP and respiratory rate, and the slight decrease in O2 saturation, are normal responses to exercise.

How well did you know this?
1
Not at all
2
3
4
5
Perfectly
59
Q

When caring for a patient who is recovering from a sudden cardiac death (SCD) event and hasno evidence of an acute myocardial infarction (AMI), the nurse will anticipate teaching the patient that
a. sudden cardiac death events rarely reoccur.
b. additional diagnostic testing will be required.
c. long-term anticoagulation therapy will be needed.
d. limiting physical activity will prevent future SCD events.

A

ANS: B
Diagnostic testing (e.g., stress test, Holter monitor, electrophysiologic studies, cardiac catheterization) is used to determine the possible cause of the SCD and treatment options. SCD is likely to recur. Anticoagulation therapy will not have any effect on the incidence of SCD, and SCD can occur even when the patient is resting

How well did you know this?
1
Not at all
2
3
4
5
Perfectly
60
Q

A few days after experiencing a myocardial infarction (MI) and successful percutaneous coronary intervention, the patient states, “It was just a little chest pain. As soon as I get out of here, I’m going for my vacation as planned.” Which reply would be most appropriate for the nurse to make?
a. “What do you think caused your chest pain?”
b. “Where are you planning to go for your vacation?”
c. “Sometimes plans need to change after a heart attack.”
d. “Recovery from a heart attack takes at least a few weeks.”

A

ANS: A
When the patient is experiencing denial, the nurse should assist the patient in testing reality until the patient has progressed beyond this step of the emotional adjustment to MI. Asking the patient about vacation plans reinforces the patient’s plan, which is not appropriate in the immediate post-MI period. Reminding the patient in denial about the MI is likely to make the patient angry and lead to distrust of the nursing staff

How well did you know this?
1
Not at all
2
3
4
5
Perfectly
61
Q

Which assessment finding by the nurse caring for a patient who has had coronary artery bypass grafting using a right radial artery graft is most important to communicate to the healthcare provider?
a. Complaints of incisional chest pain
b. Pallor and weakness of the right hand
c. Fine crackles heard at both lung bases
d. Redness on both sides of the sternal incision

A

ANS: B
The changes in the right hand indicate compromised blood flow, which requires immediate evaluation and actions such as prescribed calcium channel blockers or surgery. The other changes are expected or require nursing interventions.

How well did you know this?
1
Not at all
2
3
4
5
Perfectly
62
Q

When caring for a patient who has just arrived on the telemetry unit after having cardiac catheterization, which nursing intervention should the nurse delegate to a licensed practical/vocational nurse (LPN/LVN)?
a. Give the scheduled aspirin and lipid-lowering medication.
b. Perform the initial assessment of the catheter insertion site.
c. Teach the patient about the usual postprocedure plan of care.
d. Titrate the heparin infusion according to the agency protocol.

A

ANS: A
Administration of oral medications is within the scope of practice for LPNs/LVNs. The initial assessment of the patient, patient teaching, and titration of IV anticoagulant medications should be done by the registered nurse (RN)

How well did you know this?
1
Not at all
2
3
4
5
Perfectly
63
Q

Which electrocardiographic (ECG) change is most important for the nurse to report to the health care provider when caring for a patient with chest pain?
a. Inverted P wave
b. Sinus tachycardia
c. ST-segment elevation
d. First-degree atrioventricular block

A

ANS: C

How well did you know this?
1
Not at all
2
3
4
5
Perfectly
64
Q

When caring for a patient with acute coronary syndrome who has returned to the coronary care unit after having angioplasty with stent placement, the nurse obtains the following assessment data. Which data indicate the need for immediate action by the nurse?
a. Heart rate 102 beats/min
b. Pedal pulses 1+ bilaterally
c. Report of severe chest pain
d. Blood pressure 103/54 mm Hg

A

ANS: C
The patient’s chest pain indicates that restenosis of the coronary artery may be occurring and requires immediate actions, such as administration of oxygen and nitroglycerin, by the nurse. The other information indicates a need for ongoing assessments by the nurse

How well did you know this?
1
Not at all
2
3
4
5
Perfectly
65
Q

A patient admitted to the coronary care unit (CCU) with an ST-segment-elevation myocardial infarction (STEMI) is restless and anxious. The blood pressure is 86/40 mm Hg, and heart rateis 132 beats/min. Based on this information, which nursing diagnosis is a priority for the patient?
a. Acute pain related to myocardial infarction
b. Anxiety related to perceived threat of death
c. Stress overload related to acute change in health
d. Decreased cardiac output related to cardiogenic shock

A

ANS: D
All the nursing diagnoses may be appropriate for this patient, but the hypotension and tachycardia indicate decreased cardiac output and shock from the damaged myocardium. This will result in decreased perfusion to all vital organs (e.g., brain, kidney, heart) and is a priority.

How well did you know this?
1
Not at all
2
3
4
5
Perfectly
66
Q

When admitting a patient with a non–ST-segment-elevation myocardial infarction (NSTEMI) to the intensive care unit, which action should the nurse perform first?
a. Attach the heart monitor.
b. Obtain the blood pressure.
c. Assess the peripheral pulses.
d. Auscultate the breath sounds.

A

ANS: A
Because dysrhythmias are the most common complication of myocardial infarction (MI), the first action should be to place the patient on a heart monitor. The other actions are also important and should be accomplished as quickly as possible

How well did you know this?
1
Not at all
2
3
4
5
Perfectly
67
Q

A patient who has chest pain is admitted to the emergency department (ED), and all of the following are ordered. Which one should the nurse arrange to be completed first?
a. Chest x-ray
b. Troponin level
c. Electrocardiogram (ECG)
d. Insertion of a peripheral IV

A

ANS: C
The priority for the patient is to determine whether an acute myocardial infarction (AMI) is occurring so that the appropriate therapy can begin as quickly as possible. ECG changes occurvery rapidly after coronary artery occlusion, and an ECG should be obtained as soon as possible. Troponin levels will increase after about 3 hours. Data from the chest x-ray may impact the patient’s care but are not helpful in determining whether the patient is experiencinga myocardial infarction. Peripheral access will be needed but not before the ECG

How well did you know this?
1
Not at all
2
3
4
5
Perfectly
68
Q

After receiving change-of-shift report about the following four patients on the cardiac care unit, which patient should the nurse assess first?
a. A 39-yr-old patient with pericarditis who is complaining of sharp, stabbing chest pain
b. A 56-yr-old patient with variant angina who is scheduled to receive nifedipine (Procardia)
c. A 65-yr-old patient who had a myocardial infarction (MI) 4 days ago and is anxious about today’s planned discharge
d. A 59-yr-old patient with unstable angina who has just returned after a percutaneous coronary intervention (PCI)

A

ANS: D
After PCI, the patient is at risk for hemorrhage from the arterial access site. The nurse should assess the patient’s blood pressure, pulses, and the access site immediately. The other patients should also be assessed as quickly as possible, but assessment of this patient has the highest priority.

How well did you know this?
1
Not at all
2
3
4
5
Perfectly
69
Q

Which patient at the cardiovascular clinic requires the most immediate action by the nurse?
a. Patient with type 2 diabetes whose current blood glucose level is 145 mg/dL
b. Patient with stable angina whose chest pain has recently increased in frequency
c. Patient with familial hypercholesterolemia and a total cholesterol of 465 mg/dL
d. Patient with chronic hypertension whose blood pressure today is 172/98 mm Hg

A

ANS: B
The history of more frequent chest pain suggests that the patient may have unstable angina, which is part of the acute coronary syndrome spectrum. This will require rapid implementation of actions such as cardiac catheterization and possible percutaneous coronary intervention. The data about the other patients suggest that their conditions are stable.

How well did you know this?
1
Not at all
2
3
4
5
Perfectly
70
Q

A patient with diabetes mellitus and chronic stable angina has a new order for captopril . The nurse should teach the patient that the primary purpose of captopril is to
a. decrease the heart rate.
b. control blood glucose levels.
c. prevent changes in heart muscle.
d. reduce the frequency of chest pain

A

ANS: C
The purpose for angiotensin-converting enzyme (ACE) inhibitors in patients with chronic stable angina who are at high risk for a cardiac event is to decrease ventricular remodeling. ACE inhibitors do not directly impact angina frequency, blood glucose, or heart rate

How well did you know this?
1
Not at all
2
3
4
5
Perfectly
71
Q

To determine whether there is a delay in impulse conduction through the ventricles, the nurse will measure the duration of the patient’s
a. P wave.
b. Q wave.
c. PR interval.
d. QRS complex.

A

ANS: D
The QRS complex represents ventricular depolarization. The P wave represents the depolarization of the atria. The PR interval represents depolarization of the atria, atrioventricular node, bundle of His, bundle branches, and the Purkinje fibers. The Q wave is the first negative deflection following the P wave and should be narrow and short.

How well did you know this?
1
Not at all
2
3
4
5
Perfectly
72
Q

The nurse obtains a rhythm strip on a patient who has had a myocardial infarction and makes the following analysis: no visible P waves, PR interval not measurable, ventricular rate of 162,R-R interval regular, and QRS complex wide and distorted, and QRS duration of 0.18 second. The nurse interprets the patient’s cardiac rhythm as
a. atrial flutter.
b. sinus tachycardia.
c. ventricular fibrillation.
d. ventricular tachycardia.

A

ANS: D
The absence of P waves, wide QRS, rate greater than 150 beats/min, and the regularity of the rhythm indicate ventricular tachycardia. Atrial flutter is usually regular, has a narrow QRS configuration, and has flutter waves present representing atrial activity. Sinus tachycardia has P waves. Ventricular fibrillation is irregular and does not have a consistent QRS duration

How well did you know this?
1
Not at all
2
3
4
5
Perfectly
73
Q

The nurse notes that a patient’s heart monitor shows that every other beat is earlier than expected, has no visible P wave, and has a QRS complex that is wide and bizarre in shape. How will the nurse document the rhythm?
a. Ventricular couplets
b. Ventricular bigeminy
c. Ventricular R-on-T phenomenon
d. Multifocal premature ventricular contractions

A

ANS: B
Ventricular bigeminy describes a rhythm in which every other QRS complex is wide and bizarre looking. Pairs of wide QRS complexes are described as ventricular couplets. There is no indication that the premature ventricular contractions are multifocal or that the R-on-T phenomenon is occurring

How well did you know this?
1
Not at all
2
3
4
5
Perfectly
74
Q

A patient has a sinus rhythm and a heart rate of 72 beats/min. The nurse determines that the PR interval is 0.24 seconds. The most appropriate intervention by the nurse would be to
a. notify the health care provider immediately.
b. document the finding and monitor the patient.
c. give atropine per agency dysrhythmia protocol.
d. prepare the patient for temporary pacemaker insertion.

A

ANS: B
First-degree atrioventricular block is asymptomatic and requires ongoing monitoring because it may progress to more serious forms of heart block. The rate is normal, so there is no indication that atropine is needed. Immediate notification of the health care provider about an asymptomatic rhythm is not necessary.

How well did you know this?
1
Not at all
2
3
4
5
Perfectly
75
Q

The nurse has received change-of-shift report about the following patients on the progressive care unit. Which patient should the nurse see first?
a. A patient with atrial fibrillation, rate 88 and irregular, who has a dose of warfarin (Coumadin) due
b. A patient with second-degree atrioventricular (AV) block, type 1, rate 60, who is dizzy when ambulating
c. A patient who is in a sinus rhythm, rate 98 and regular, recovering from an
elective cardioversion 2 hours ago
d. A patient whose implantable cardioverter-defibrillator (ICD) fired twice today and has a dose of amiodarone (Cordarone) due

A

ANS: D
The frequent firing of the ICD indicates that the patient’s ventricles are very irritable and the priority is to assess the patient and give the amiodarone. The other patients can be seen after the amiodarone is given

How well did you know this?
1
Not at all
2
3
4
5
Perfectly
76
Q

A patient who is on the telemetry unit develops atrial flutter, rate 150, with associated dyspneaand chest pain. Which action that is included in the hospital dysrhythmia protocol should the nurse do first?
a. Obtain a 12-lead electrocardiogram (ECG).
b. Notify the health care provider of the change in rhythm.
c. Give supplemental O2 at 2 to 3 L/min via nasal cannula.
d. Assess the patient’s vital signs including O2 saturation.

A

ANS: C
Because this patient has dyspnea and chest pain in association with the new rhythm, the nurse’s initial actions should be to address the patient’s airway, breathing, and circulation (ABC) by starting with O2 administration. The other actions are also important and should be implemented rapidly

How well did you know this?
1
Not at all
2
3
4
5
Perfectly
77
Q

To determine whether there is a delay in impulse conduction through the atria, the nurse will measure the duration of the patients

a. P wave.
b. Q wave.
c. P-R interval.
d. QRS complex.

A

ANS: A
The P wave represents the depolarization of the atria. The P-R interval represents depolarization of the atria, atrioventricular (AV) node, bundle of His, bundle branches, and the Purkinje fibers. The QRS represents ventricular depolarization. The Q wave is the first negative deflection following the P wave and should be narrow and short.

How well did you know this?
1
Not at all
2
3
4
5
Perfectly
78
Q

The home health nurse is visiting a patient with chronic obstructive pulmonary disease (COPD). Which nursing action is appropriate to implement for a nursing diagnosis of impaired breathing pattern related to anxiety?
a. Titrate O2 to keep saturation at least 90%.
b. Teach the patient how to use pursed-lip breathing.
c. Discuss a high-protein, high-calorie diet with the patient.
d. Suggest the use of over-the-counter sedative medications.

A

ANS: B
Pursed-lip breathing techniques assist in prolonging the expiratory phase of respiration and decrease air trapping. There is no indication that the patient requires O2 therapy or an improved diet. Sedative medications should be avoided because they decrease respiratory drive

How well did you know this?
1
Not at all
2
3
4
5
Perfectly
79
Q

A patient with chronic obstructive pulmonary disease (COPD) has a nursing diagnosis of imbalanced nutrition: less than body requirements. Which intervention would be most appropriate for the nurse to include in the plan of care?
a. Encourage increased intake of whole grains.
b. Increase the patient’s intake of fruits and fruit juices.
c. Offer high-calorie protein snacks between meals and at bedtime.
d. Assist the patient in choosing foods with high vegetable content.

A

ANS: C
Eating small amounts more frequently (as occurs with snacking) will increase caloric intake by decreasing the fatigue and feelings of fullness associated with large meals. Patients with COPD should rest before meals. Foods that have a lot of texture such as whole grains may take more energy to eat and get absorbed and lead to decreased intake. Although fruits, juices, and minerals are not contraindicated, foods high in protein are a better choice

How well did you know this?
1
Not at all
2
3
4
5
Perfectly
80
Q

The nurse interviews a patient with a new diagnosis of chronic obstructive pulmonary disease (COPD). Which information is most specific in confirming a diagnosis of chronic bronchitis?
a. The patient tells the nurse about a family history of bronchitis.
b. The patient indicates a 30 pack-year cigarette smoking history.
c. The patient reports a productive cough for 3 months every winter.
d. The patient denies having respiratory problems until the past 12 months.

A

ANS: C
A diagnosis of chronic bronchitis is based on a history of having a productive cough for 3 months for at least 2 consecutive years. There is no family tendency for chronic bronchitis. Although smoking is the major risk factor for chronic bronchitis, a smoking history does not confirm the diagnosis

How well did you know this?
1
Not at all
2
3
4
5
Perfectly
81
Q

A patient with chronic obstructive pulmonary disease (COPD) has poor gas exchange. Which action by the nurse would support the patient’s ventilation?
a. Have the patient rest in bed with the head elevated to 15 to 20 degrees.
b. Encourage the patient to sit up at the bedside in a chair and lean forward.
c. Ask the patient to rest in bed in a high-Fowler’s position with the knees flexed.
d. Place the patient in the Trendelenburg position with pillows behind the head.

A

ANS: B
Patients with COPD improve the mechanics of breathing by sitting up in the “tripod” position.Resting in bed with the head elevated in a semi-Fowler’s position would be an alternative position if the patient was confined to bed, but sitting in a chair allows better ventilation. The Trendelenburg position or sitting upright in bed with the knees flexed would decrease the patient’s ability to ventilate well.

How well did you know this?
1
Not at all
2
3
4
5
Perfectly
82
Q

The nurse in the emergency department receives arterial blood gas results for four recently admitted patients with obstructive pulmonary disease. The results for which patient will require the most rapid action by the nurse?
a. pH 7.28, PaCO2 50 mm Hg, and PaO2 58 mm Hg
b. pH 7.48, PaCO2 30 mm Hg, and PaO2 65 mm Hg
c. pH 7.34, PaCO2 33 mm Hg, and PaO2 80 mm Hg
d. pH 7.31, PaCO2 58 mm Hg, and PaO2 64 mm Hg

A

ANS: A
The pH, PaCO2, and PaO2 indicate that the patient has severe uncompensated respiratory acidosis and hypoxemia. Rapid action will be required to prevent increasing hypoxemia and correct the acidosis

How well did you know this?
1
Not at all
2
3
4
5
Perfectly
83
Q

Which nursing action for a patient with chronic obstructive pulmonary disease (COPD) could the nurse delegate to experienced unlicensed assistive personnel (UAP)?
a. Obtain O2 saturation using pulse oximetry.
b. Monitor for increased O2 need with exercise.
c. Teach the patient about safe use of O2 at home.
d. Adjust O2 to keep saturation in prescribed parameters.

A

ANS: A
UAP can obtain O2 saturation (after being trained and evaluated in the skill). The other actionsrequire more education and a scope of practice that licensed practical/vocational nurses (LPN/LVNs) or registered nurses (RNs) would have

How well did you know this?
1
Not at all
2
3
4
5
Perfectly
84
Q

The nurse reviews the medication administration record (MAR) for a patient having an acute asthma attack. Which medication should the nurse administer first?
a. Methylprednisolone (Solu-Medrol) 60 mg IV
b. Albuterol (Ventolin HFA) 2.5 mg per nebulizer
c. Salmeterol (Serevent) 50 mcg per dry-powder inhaler (DPI)
d. Ipratropium (Atrovent) 2 puffs per metered-dose inhaler (MDI)

A

ANS: B
Albuterol is a rapidly acting bronchodilator and is the first-line medication to reverse airway narrowing in acute asthma attacks. The other medications work more slowly.

How well did you know this?
1
Not at all
2
3
4
5
Perfectly
85
Q

The nurse receives a change-of-shift report on the following patients with chronic obstructive pulmonary disease (COPD). Which patient should the nurse assess first?
a. A patient with loud expiratory wheezes
b. A patient with a respiratory rate of 38 breaths/min
c. A patient who has a cough productive of thick, green mucus
d. A patient with jugular venous distention and peripheral edema

A

ANS: B

How well did you know this?
1
Not at all
2
3
4
5
Perfectly
86
Q

On admission of a patient to the postanesthesia care unit (PACU), the blood pressure (BP) is 122/72 mm Hg. Thirty minutes after admission, the BP is 114/62, with a pulse of 74 and warm, dry skin. Which action by the nurse is most appropriate?
a. Increase the IV fluid rate.
b. Notify the anesthesia care provider (ACP).
c. Continue to take vital signs every 15 minutes.
d. Administer oxygen therapy at 100% per mask.

A

ANS: C
A slight drop in postoperative BP with a normal pulse and warm, dry skin indicates normal response to the residual effects of anesthesia and requires only ongoing monitoring. Hypotension with tachycardia or cool, clammy skin would suggest hypovolemic or hemorrhagic shock and the need for notification of the ACP, increased fluids, and high-concentration oxygen administration

How well did you know this?
1
Not at all
2
3
4
5
Perfectly
87
Q

In the postanesthesia care unit (PACU), a patient’s vital signs are blood pressure 116/72 mm Hg, pulse 74 beats/min, respirations 12 breaths/min, and SpO2 91%. The patient is sleepy but awakens easily. Which action should the nurse take first?
a. Place the patient in a side-lying position.
b. Encourage the patient to take deep breaths.
c. Prepare to transfer the patient to a clinical unit.
d. Increase the rate of the postoperative IV fluids.

A

ANS: B
The patient’s borderline SpO2 and sleepiness indicate hypoventilation. The nurse should stimulate the patient and remind the patient to take deep breaths. Placing the patient in a lateral position is needed when the patient first arrives in the PACU and is unconscious. The stable blood pressure and pulse indicate that no changes in fluid intake are required. The patient is not fully awake and has a low SpO2, indicating that transfer from the PACU to a clinical unit is not appropriate

88
Q

An older patient is being discharged from the ambulatory surgical unit following left eye surgery. The patient tells the nurse, “I don’t know if I can take care of myself once I’m home.”Which action by the nurse is most appropriate?
a. Provide written instructions for the care.
b. Assess the patient’s home support system.
c. Discuss specific concerns regarding self-care.
d. Refer the patient for home health care services.

A

ANS: C
The nurse’s initial action should be to assess exactly the patient’s concerns about self-care. Referral to home health care and assessment of the patient’s support system may be appropriate actions but will be based on further assessment of the patient’s concerns. Written instructions should be given to the patient, but these are unlikely to address the patient’s statedconcern about self-care.

89
Q

The nasogastric (NG) tube is removed on the second postoperative day, and the patient is placed on a clear liquid diet. Four hours later, the patient complains of frequent, cramping gas pains. What action by the nurse is the most appropriate?
a. Reinsert the NG tube.
b. Give the PRN IV opioid.
c. Assist the patient to ambulate.
d. Place the patient on NPO status.

A

ANS: C
Ambulation encourages peristalsis and the passing of flatus, which will relieve the patient’s discomfort. If distention persists, the patient may need to be placed on NPO status, but usuallythis is not necessary. Morphine administration will further decrease intestinal motility. Gas pains are usually caused by trapping of flatus in the colon, and reinsertion of the NG tube will not relieve the pains

90
Q

A patient’s T-tube is draining dark green fluid after gallbladder surgery. What action by the nurse is the most appropriate?
a. Notify the patient’s surgeon.
b. Place the patient on bed rest.
c. Document the color and amount of drainage.
d. Irrigate the T-tube with sterile normal saline.

A

ANS: C
A T-tube normally drains dark green to bright yellow drainage so no action other than to document the amount and color of the drainage is needed. The other actions are not necessary

91
Q

Which action by the nurse will be most helpful to a patient who is expected to ambulate, deep breathe, and cough on the first postoperative day?
a. Schedule the activity to begin after the patient has taken a nap.
b. Administer prescribed analgesic medications before the activities.
c. Ask the patient to state two possible complications of immobility.
d. Encourage the patient to state the purpose of splinting the incision.

A

ANS: B
An important nursing action to encourage these postoperative activities is administration of adequate analgesia to allow the patient to accomplish the activities with minimal pain. Even with motivation provided by proper teaching, positive reinforcement, concern about complications, and with rest and sleep, patients will have difficulty if there is a great deal of pain involved with these activities.

92
Q

A postoperative patient has a nursing diagnosis of ineffective airway clearance. The nurse determines that interventions for this nursing diagnosis have been successful if which is observed?
a. Patient drinks 2 to 3 L of fluid in 24 hours.
b. Patient uses the spirometer 10 times every hour.
c. Patient’s breath sounds are clear to auscultation.
d. Patient’s temperature is less than 100.2°F orally.

A

ANS: C
One characteristic of ineffective airway clearance is the presence of adventitious breath sounds such as crackles, so clear breath sounds are an indication of resolution of the problem. Spirometer use and increased fluid intake are interventions for ineffective airway clearance but may not always improve breath sounds. Elevated temperature may occur with atelectasis, but a normal or near-normal temperature does not always indicate resolution of respiratory problems

93
Q

Which action could the postanesthesia care unit (PACU) nurse delegate to unlicensed assistivepersonnel (UAP) who help to transport a patient to the clinical unit?
a. Clarify the postoperative orders with the surgeon.
b. Help with the transfer of the patient onto a stretcher.
c. Document the appearance of the patient’s incision in the chart.
d. Provide hand off communication to the surgical unit charge nurse.

A

ANS: B
The scope of practice of UAP includes repositioning and moving patients under the supervision of a nurse. Providing report to another nurse, assessing and documenting the wound appearance, and clarifying physician orders with another nurse require registered-nurse(RN) level education and scope of practice.

94
Q

A patient is transferred from the postanesthesia care unit (PACU) to the clinical unit. Which action by the nurse on the clinical unit should be performed first?
a. Assess the patient’s pain.
b. Orient the patient to the unit.
c. Take the patient’s vital signs.
d. Read the postoperative orders

A

ANS: C
Because the priority concerns after surgery are airway, breathing, and circulation, the vital signs are assessed first. The other actions should take place after the vital signs are obtained and compared with the vital signs before transfer

95
Q

An older patient who had knee replacement surgery 2 days ago can only tolerate being out of bed with physical therapy twice a day. Which collaborative problem should the nurse identify as a priority for this patient?
a. Potential complication: hypovolemic shock
b. Potential complication: venous thromboembolism
c. Potential complication: fluid and electrolyte imbalance
d. Potential complication: impaired surgical wound healing

A

ANS: B
The patient is older and relatively immobile, which are two risk factors for development of deep vein thrombosis. The other potential complications are possible postoperative problems, but they are not at a high risk based on the data about this patient

96
Q

A patient who is just waking up after having hip replacement surgery is agitated and confused.Which action should the nurse take first?
a. Administer the prescribed opioid.
b. Check the oxygen (O2) saturation.
c. Take the blood pressure and pulse.
d. Apply wrist restraints to secure IV lines.

A

ANS: B
Emergence delirium may be caused by a variety of factors. However, the nurse should first assess for hypoxemia. The other actions also may be appropriate, but are not the best initial action

97
Q

The nurse is caring for a patient the first postoperative day following a laparotomy for a small bowel obstruction. The nurse notices new bright-red drainage about 5 cm in diameter on the dressing. Which action should the nurse take first?
a. Reinforce the dressing.
b. Apply an abdominal binder.
c. Take the patient’s vital signs.
d. Recheck the dressing in 1 hour.

A

ANS: C
New bright-red drainage may indicate hemorrhage, and the nurse should initially assess the patient’s vital signs for tachycardia and hypotension. The surgeon should then be notified of the drainage and the vital signs. The dressing may be changed or reinforced, based on the surgeon’s orders or agency policy. The nurse should not wait an hour to recheck the dressing.

98
Q

The nurse assesses that the oxygen saturation is 89% in an unconscious patient who was transferred from surgery to the postanesthesia care unit (PACU) 15 minutes ago. Which actionshould the nurse take first?
a. Suction the patient’s mouth.
b. Increase the oxygen flow rate.
c. Perform the jaw-thrust maneuver.
d. Elevate the patient’s head on two pillows.

A

ANS: C
In an unconscious postoperative patient, a likely cause of hypoxemia is airway obstruction by the tongue, and the first action is to clear the airway by maneuvers such as the jaw thrust or chin lift. Increasing the oxygen flow rate and suctioning are not helpful when the airway is obstructed by the tongue. Elevating the patient’s head will not be effective in correcting the obstruction but may help with oxygenation after the patient is awake

99
Q

Which finding would indicate to the nurse that a postoperative patient is at increased risk for poor wound healing?
a. Potassium 3.5 mEq/L
b. Albumin level 2.2 g/dL
c. Hemoglobin 10.2 g/dL
d. White blood cells 11,900/µL

A

ANS: B
Because proteins are needed for an appropriate inflammatory response and wound healing, thelow serum albumin level (normal level, 3.5 to 5.0 g/dL) indicates a risk for poor wound healing. The potassium level is normal. Because a small amount of blood loss is expected withsurgery, the hemoglobin level is not indicative of an increased risk for wound healing. WBC count is expected to increase after surgery as a part of the normal inflammatory response

100
Q

The nurse assesses a patient on the second postoperative day after abdominal surgery to repaira perforated duodenal ulcer. Which finding is most important for the nurse to report to the surgeon?
a. Tympanic temperature 99.2° F (37.3° C)
b. Fine crackles audible at both lung bases
c. Redness and swelling along the suture line
d. 200 mL sanguineous fluid in the wound drain

A

ANS: D
Wound drainage should decrease and change in color from sanguineous to serosanguineous bythe second postoperative day. The color and amount of drainage for this patient are abnormal and should be reported. Redness and swelling along the suture line and a slightly elevated temperature are normal signs of postoperative inflammation. Atelectasis is common after surgery. The nurse should have the patient cough and deep breathe, but there is no urgent needto notify the surgeon.

101
Q

After receiving change-of-shift report about these postoperative patients, which patient shouldthe nurse assess first?
a. Obese patient who had abdominal surgery 3 days ago and whose wound edges are separating
b. Patient who has 30 mL of sanguineous drainage in the wound drain 10 hours after hip replacement surgery
c. Patient who has bibasilar crackles and a temperature of 100° F (37.8 °C) on the first postoperative day after chest surgery
d. Patient who continues to have incisional pain 15 minutes after hydrocodone and acetaminophen (Vicodin) was given

A

ANS: A
The patient’s history and assessment suggests possible wound dehiscence, which should be reported immediately to the surgeon. Although the information about the other patients indicates a need for ongoing assessment and possible intervention, the data do not suggest anyacute complications. Small amounts of red drainage are common in the first postoperative hours. Bibasilar crackles and a slightly elevated temperature are common after surgery, although the nurse will need to have the patient deep breathe and cough. Oral medications typically take more than 15 minutes for effective pain relief

102
Q

A patient’s blood pressure in the postanesthesia care unit (PACU) has dropped from an admission blood pressure of 140/86 to 102/60 mm Hg with a pulse change of 70 to 96 beats/min. SpO2 is 92% on 3 L of oxygen. In which order should the nurse take these actions? (Put a comma and a space between each answer choice [A, B, C, D].)
a. Increase the IV infusion rate.
b. Assess the patient’s dressing.
c. Increase the oxygen flow rate.
d. Check the patient’s temperature.

A

ANS:A, C, B, D
The first nursing action should be to increase the IV infusion rate. Because the most common cause of hypotension is volume loss, the IV rate should be increased. The next action should be to increase the oxygen flow rate to maximize oxygenation of hypoperfused organs. Because hemorrhage is a common cause of postoperative volume loss, the nurse should check the dressing. Finally, the patient’s temperature should be assessed to determine the effects of vasodilation caused by rewarming.

103
Q

The nurse is teaching a group of older adults about risk factors related to hip fractures. Which information should the nurse include in the​ presentation? (Select all that​ apply.)
A. Arthritis
B. Lack of physical activity
C. Osteoporosis
D. Tobacco use
E.Calcium deficiency

A

ANS: ​B, C, D, E
Rationale: Risk factors for hip fractures include lack of physical​ activity; deficiency in calcium or vitamin​ D; tobacco and alcohol​ use; and osteoporosis. Arthritis is not considered a risk factor for hip fractures.

104
Q

The nurse is assessing a​ client’s risk for sustaining a hip fracture. Which information should the nurse obtain when obtaining the health​ history? (Select all that​ apply.)
A. History of osteoporosis
B. Skin integrity
C. Age
D. Vital signs
E. History of falls

A

ANS: A, C, E
​Rationale: The health history of a client with a hip fracture should include​ age, history of​ falls, and history of osteoporosis. Vital signs and skin integrity are obtained when performing a physical examination.

105
Q

The nurse is assigned to care for a client who experienced a recent fall. Which manifestation indicates that the​ client’s hip is​ fractured?
A. Complaints of stiffness when transferring to chair
B. The affected leg is shorter than the other and turned outward
C. Bruising noted to the injured hip and leg
D. Discomfort when performing range of motion exercises

A

ANS: B
​Rationale: The leg of the injured hip is shorter than the uninjured leg and is sometimes turned outward in clients with hip fracture. These clients complain of severe​ pain, not​ discomfort, when flexing and rotating the hip. Bruising noted to the hip and leg may or may not be related to the fall. Complaints of stiffness may be related to the fall or from lying in bed.

106
Q

The nurse is caring for four clients. Which client should the nurse identify as having the highest risk for sustaining a hip fracture if they sustain a​ fall?
A. 60-year-old man admitted for treatment of pneumonia
B. 80-year-old man admitted for benign prostatic hypertrophy
C. ​50-year-old woman with a history of osteoarthritis
D. 70-year-old woman who consumes 800 mg​ calcium/day

A

ANS: D
​Rationale: Women who are postmenopausal and not taking estrogen should consume a minimum of 1500 mg of calcium per day to maintain bone health. The​ 70-year-old woman who only consumes 800 mg of calcium per day is at the highest risk for a hip fracture if she falls. The​ 50-year-old woman may not be postmenopausal and is at a lower​ risk, and the men are at a lower risk.

107
Q

The nurse is teaching an older adult client about preventing hip fractures. Which information should the nurse​ include? (Select all that​ apply.)
A. Obtaining a screening to test for osteoporosis
B. Maintaining adequate intake of calcium and vitamin D
C. Ensuring throw rugs are placed throughout the home
D. Performing​ weight-bearing exercises daily
E. Drinking one glass of red wine every night

A

ANS: A, B, D
​Rationale: Teaching the client about avoiding falls can be helpful in preventing hip fractures.​ Weight-bearing exercises increase strength and adequate intake of calcium and vitamin D helps bone health. Screening for osteoporosis can lead to early treatment to help diminish the risk of bone fractures. Throw rugs are not recommended because the client can trip or slip on them. There is no recommendation to drink red​ wine; in​ fact, alcohol should be consumed with caution as it can impair balance and increase the risk for a fall.

108
Q

The home care nurse is visiting an older adult client with a new diagnosis of macular degeneration and decreased visual acuity. Which instruction should the nurse provide the caregiver to decrease the​ client’s risk of sustaining a fall and a hip​ fracture? (Select all that​ apply.)
A. Remove throw rugs
B. Clear pathways
C. Eliminate alcohol
D. Use​ night-lights
E. Increase calcium

A

Answer: A, B, D
​Rationale: An older adult client with decreased visual acuity is at high risk for falling.​ Therefore, the nurse would instruct the family to clear the​ pathways, use​ night-lights, and remove throw rugs. Calcium should be increased for postmenopausal women. Excessive alcohol intake should be avoided.

109
Q

The nurse is caring for an​ 8-year-old child who sustained a hip fracture from a motor vehicle crash. The parents ask if the child will be scheduled for a hip replacement. How should the nurse​ respond?
A. “Treatment for hip fractures in children often involves casting for 4 to 6​ weeks.”
B. ​”Hip replacements are not done in children because they need to be revised with​ growth.”
C. “We will place the child in traction for a few​ days, then do the hip​ replacement.”
D. “A hip replacement will be performed once the child is medically​ stable.”

A

Answer: A
​Rationale: Hip fractures in children are usually treated with casting for 4 to 6 weeks or repair surgery rather than hip replacement surgery. Hip replacements begin to fail at 10 years and require revision surgery.

110
Q

An older adult client sustained a hip fracture secondary to a fall and undergoes an arthroplasty. The client refuses to get out of bed due to pain and fatigue. Which response by the nurse is​ correct?
A. “It is okay to rest​ today, but you need to participate​ tomorrow.”
B. “You have to get out of bed today because the healthcare provider ordered​ it.”
C. ​”Early ambulation promotes healing and reduces​ complications.”
D. “We will give you pain medication after you get up and participate in​ therapy.”

A

Answer: C
​Rationale: The nurse should inform the client that getting out of bed the first postoperative day will decrease complications and improve mobility. The nurse would not tell the client it is okay to wait one day. The nurse would not inform the client that they need to get out of bed because the healthcare provider ordered it. This is not therapeutic communication and does not provide information. Pain medication should be administered prior to​ therapy, not after.

111
Q

Which information should the nurse provide a​ 70-year-old client to prevent falls and hip​ fractures? (Select all that​ apply.)
A. Having an eye exam every year
B. Limiting cigarette smoking
C. Taking 500 mg of calcium every day
D. Participating in​ weight-bearing exercises
E. Avoiding excessive alcohol use

A

Answer: A, D, E
​Rationale: Yearly eye​ exams, daily​ weight-bearing exercises, and avoiding excessive alcohol use are interventions to help reduce falls and prevent hip fractures. Any amount of cigarette smoking places a client at risk of hip​ fractures; the client needs to refrain from smoking altogether. A postmenopausal woman who is not on estrogen replacement should take​ 1,500 mg of calcium daily.

112
Q

Which pharmacologic treatment should the nurse expect the healthcare provider to prescribe to a client with osteoporosis to prevent hip​ fractures?
A. Selective serotonin reuptake inhibitors​ (SSRIs)
B. Loop diuretics
C. Incretin mimetic agents
D. Bone density enhancers

A

Answer: D
​Rationale: Bone density enhancers​ (e.g., bisphosphonates) stimulate bone growth and can be administered to clients with osteoporosis to prevent hip fractures. Incretin mimetic​ agents, SSRIs, and loop diuretics are not indicated in the treatment of osteoporosis to prevent hip fractures.

113
Q

The nurse is caring for a client following the surgical repair of a hip fracture. Which intervention assists in reducing the risk of a deep vein thrombosis​ (DVT)? (Select all that​ apply.)
A. Positioning an abduction pillow between the legs
B. Administering anticoagulants as prescribed
C. Using an incentive spirometer every hour
D. Placing compression stockings on the client
E. Turning the client every 2 hours

A

Answer: B, D
​Rationale: To reduce the risk of a​ DVT, administer anticoagulants as prescribed and place compression stockings on the client. Using an incentive spirometer reduces the risk of pneumonia. Turning the client every 2 hours prevents skin breakdown. Positioning an abduction pillow between the legs keeps the surgical hip in alignment.

114
Q

The nurse is preparing to send the client who is​ one-day postoperative from a hip arthroplasty for physical therapy. Which intervention should the nurse perform ​first?
A. Administer analgesics
B. Provide the client lunch
C. Administer a diuretic
D. Apply sequential compression stockings

A

Answer: A
Rationale: The nurse should administer analgesics about​ 30-60 minutes prior to attending physical therapy. This minimizes pain during exercise and allows better movement. The nurse would not administer a diuretic prior to going to therapy because the client would have to urinate frequently. The client can eat​ lunch, but it is not a priority. Sequential stockings can only be used while the client is in bed.

115
Q

Which statement accurately describes anemia?
a. Increased blood viscosity
b. Depressed hematopoietic system
c. Presence of abnormal hemoglobin
d. Decreased oxygen-carrying capacity of blood

A

ANS: D
Anemia is a condition in which the number of red blood cells or hemoglobin concentration is reduced below the normal values for age. This results in blood having a decreased oxygen-carrying capacity. Increased blood viscosity is usually a function of too many cells or dehydration, not anemia. A depressed hematopoietic system or abnormal hemoglobin can contribute to anemia, but the definition depends on the decreased oxygen-carrying capacity of the blood.

116
Q

Which statement best describes the pathological changes of sickle cell anemia?
a. Sickle-shaped cells carry excess oxygen.
b. Sickle-shaped cells decrease blood viscosity.
c. Increased red blood cell destruction occurs.
d. Decreased red blood cell destruction occurs.

A

ANS: C
The clinical features of sickle cell anemia are primarily the result of increased red blood cell destruction and obstruction caused by the sickle-shaped red blood cells. Sickled red cells have decreased oxygen-carrying capacity and transform into the sickle shape in conditions of low oxygen tension. When the sickle cells change shape, they increase the viscosity in the area where they are involved in microcirculation

117
Q

Which clinical manifestation should the nurse expect when a child with sickle cell anemia experiences an acute vaso-occlusive crisis?
a. Circulatory collapse
b. Cardiomegaly, systolic murmurs
c. Hepatomegaly, intrahepatic cholestasis
d. Painful swelling of hands and feet, painful joints

A

ANS: D
A vaso-occlusive crisis is characterized by severe pain in the area of involvement. If in the extremities, painful swelling of the hands and feet is seen; if in the abdomen, severe pain resembles that of acute surgical pain in the abdomen; and if in the head, stroke and visual disturbances occur. Circulatory collapse results from sequestration crises. Cardiomegaly, systolic murmurs, hepatomegaly, and intrahepatic cholestasis result from chronic vaso-occlusive phenomena

118
Q

A school-age child is admitted in vaso-occlusive sickle cell crisis. What should the child’s care include?
a. Correction of acidosis
b. Adequate hydration and pain management
c. Pain management and administration of heparin
d. Adequate oxygenation and replacement of factor VIII

A

ANS: B
The management of crises includes adequate hydration, minimizing energy expenditures, pain management, electrolyte replacement, and blood component therapy if indicated. The acidosis will be corrected as the crisis is treated. Heparin and factor VIII are not indicated in the treatment of a vaso-occlusive sickle cell crisis. Oxygen may prevent further sickling, but it is not effective in reversing sickling because it cannot reach the clogged blood vessels.

119
Q

Which statement about how sickle cell anemia is passed to offspring is CORRECT?*
A. This disease is an x-linked recessive disease.
B. Sickle cell anemia is an autosomal dominant disease.
C. This condition is an autosomal recessive disease.
D. Sickle cell anemia is rarely passed to offspring and is an autosomal x-linked dominant disease.

A

ANS: C
SCA is an autosomal recessive disease in that the offspring must receive TWO hemoglobin S genes (one for each parent). The parents usually don’t have the disease but are carriers. For the disease to occur in the offspring they must receive both of those genes (Hbg SS). On the contrary, with autosomal dominant the offspring has to only receive an abnormal gene from one parent, who probably has signs and symptoms of the disease too.

120
Q

Which type of hemoglobin is present in a patient who has sickle cell anemia?
A. Hemoglobin AA
B. Hemoglobin AS
C. Hemoglobin SS
D. Hemoglobin AC

A

ANS: C.
SCA is homozygous and the patient must have two abnormal alleles present to have sickle cell anemia. The patient receives each abnormal allele for each parent (hence one from each parent which is Hemoglobin SS). If a patient has Hemoglobin AS (normal allele (A) and abnormal allele (S)) this is known as sickle cell trait, which most patients with this don’t present with signs and symptoms of the disease…it’s rare because they usually have just enough hemoglobin A to prevent the RBCs from sickling.

121
Q

A 25 year-old pregnant female and her partner both have sickle cell trait. What is the percentage that their offspring will develop sickle cell anemia?
A. 50%
B. 25%
C. 75%
D. 100%

A

ANS: B
If both parents have the sickle cell trait it means they each have normal hemoglobin A and abnormal hemoglobin S on their RBCs….so both present with hbg AS. Remember they don’t have sickle cell disease just the abnormal gene that can be passed to their child. Sickle cell anemia is autosomal recessive, therefore there is a 25% chance their child will obtain both abnormal genes (the Hbg S) from EACH parent and develop sickle cell anemia.

122
Q

You’re assisting a physician with sickle cell anemia screening. As the nurse you know that which patient population listed below is at risk for sickle cell disease?
A. Native Americans
B. African-Americans
C. Pacific Islanders
D. Latino

A

ANS: B. Sickle cell anemia is most common in African-Americans along with Middle Eastern, Asian, Caribbean, and Eastern Mediterranean.

123
Q

A 14 year-old female has sickle cell anemia. Which factors below can increase the patient’s risk for developing sickle cell crisis? Select all that apply:
A. Shellfish
B. Infection
C. Dehydration
D. Hypoxia
E. Low altitudes
F. Hemorrhage
G. Strenuous exercise

A

ANS: B, C, D, F and G.
Sickle cell crisis can occur when the body experiences low amounts of oxygen in the body (so think about something that increases the body’s need for oxygen or affects how oxygen is being transported). Therefore, infection (especially respiratory infections), dehydration, hypoxia, HIGH (not low) altitudes, hemorrhage (blood loss), or strenuous exercise can lead to a sickle cell crisis.

124
Q

A 6 year-old is admitted with sickle cell crisis. The patient has a FACE scale rating of 10 and the following vital signs: HR 115, BP 120/82, RR 18, oxygen saturation 91%, temperature 101.4’F. Select all the appropriate nursing interventions for this patient at this time?
A. Administer IV Morphine per MD order
B. Administer oxygen per MD order
C. Keep NPO
D. Apply cold compresses
E. Start intravenous fluids per MD order
F. Administer iron supplement per MD order
G. Keep patient on bed rest
H. Remove restrictive clothing or objects from the patient

A

ANS: A, B, E, G, and H.
When a patient is in sickle cell crisis, the abnormal RBCs are sickling and sticking together, which blocks blood flow. To help alleviate the RBCs from clumping together and sickling, oxygen and hydration are priority. This will help dilute the blood (hence decrease the sticking of RBCs) and help supply oxygen to the RBCs (remember abnormal RBCs with hemoglobin S are very sensitive to low oxygen levels and will sickle when there is low oxygen). In addition, pain needs to be addressed. Opioid medication is the best on a scheduled basis rather than PRN (as needed). Avoid keeping patient NPO unless needed (remember patient needs hydration). Avoid cold compresses (can lead to more sickling) but instead use warm compresses. The patient will need FOLIC ACID supplements to help with RBC creation rather than iron (iron can actually build up in the body and collect in the organs in patients with sickle cell disease). Patients definitely need to be on bedrest, and restrictive clothing or objects (blood pressure cuff etc.) should be removed to help blood flow.

125
Q

During an outpatient well visit with a patient who has sickle cell anemia, you make it PRIORITY to assess the patient’s?
A. hemoglobin A1C level
B. heart rate
C. reflexes
D. vaccination history

A

ANS: D
Patients will sickle cell anemia are at risk for infection because of spleen compromise. Many patients with SCA experience splenomegaly because blood flow is compromised to the spleen due to sickling of RBCs and the spleen is overworked from recycling the old RBCs (remember a patient with sickle cell anemia does NOT have long-living RBCs…the RBCs tend to die in 20 days rather than 120 days). Therefore, vaccination history is very important. The patient should be up-to-date with the flu, pneumococcal, and meningococcal vaccines.

126
Q

You’re providing seminar teaching to a group of nurses about sickle cell anemia. Which of the following is NOT a treatment for this condition?*
A. Blood transfusion
B. Stem cell transplant
C. Intravenous fluids
D. Iron supplements
E. Antibiotics
F. Morphine

A

ANS: B
This medication can lower the white blood cell count. Therefore, the nurse should make it priority to tell the patient to avoid infection by avoiding sick people and performing hand hygiene regularly.

127
Q

The nurse is advising a clinic patient who was exposed a week ago to human immunodeficiency virus (HIV) through unprotected sexual intercourse. The patient’s antigen and antibody test has just been reported as negative for HIV. What instructions should the nurse give to this patient?
a. “You will need to be retested in 2 weeks.”
b. “You do not need to fear infecting others.”
c. “Since you don’t have symptoms and you have had a negative test, you do not have HIV).”
d. “We won’t know for years if you will develop acquired immunodeficiency syndrome (AIDS).”

A

ANS: A
HIV screening tests detect HIV-specific antibodies or antigens, but typically it takes a several week delay after initial infection before HIV can be detected on a screening test. Combination antibody and antigen tests (also known as fourth-generation tests) decrease the window periodto within 3 weeks after infection. It is not known based on this information whether the patientis infected with HIV or can infect others

128
Q

patient informed of a positive rapid antibody test result for human immunodeficiency virus (HIV) is anxious and does not appear to hear what the nurse is saying. What action by the nurse is most important at this time?
a. Teach the patient how to reduce risky behaviors.
b. Inform the patient about the available treatments.
c. Remind the patient about the need to return for retesting to verify the results.
d. Ask the patient to identify individuals who had intimate contact with the patient.

A

ANS: C
After an initial positive antibody test result, the next step is retesting to confirm the results. A patient who is anxious is not likely to be able to take in new information or be willing to disclose information about the HIV status of other individuals

129
Q

Which patient exposure by the nurse is most likely to require postexposure prophylaxis when the patient’s human immunodeficiency virus (HIV) status is unknown?
a. Needle stick injury with a suture needle during a surgery
b. Splash into the eyes while emptying a bedpan containing stool
c. Needle stick with a needle and syringe used for a venipuncture
d. Contamination of open skin lesions with patient vaginal secretions

A

ANS: C

130
Q

The nurse palpates enlarged cervical lymph nodes on a patient diagnosed with acute human immunodeficiency virus (HIV) infection. Which action would be appropriate for the nurse to take?
a. Instruct the patient to apply ice to the neck.
b. Explain to the patient that this is an expected finding.
c. Request that an antibiotic be prescribed for the patient.
d. Advise the patient that this indicates influenza infection.

A

ANS: B
Persistent generalized lymphadenopathy is common in the early stages of HIV infection. No antibiotic is needed because the enlarged nodes are probably not caused by bacteria. Lymphadenopathy is common with acute HIV infection and is therefore not likely the flu. Ice will not decrease the swelling in persistent generalized lymphadenopathy

131
Q

Which nursing action will be most useful in assisting a college student to adhere to a newly prescribed antiretroviral therapy (ART) regimen?
a. Give the patient detailed information about possible medication side effects.
b. Remind the patient of the importance of taking the medications as scheduled.
c. Encourage the patient to join a support group for students who are HIV positive.
d. Check the patient’s class schedule to help decide when the drugs should be taken.

A

ANS: D
The best approach to improve adherence is to learn about important activities in the patient’s life and adjust the ART around those activities. The other actions are also useful, but they will not improve adherence as much as individualizing the ART to the patient’s schedule

132
Q

A patient with human immunodeficiency virus (HIV) infection has developed Mycobacteriumavium complex infection. Which outcome would be appropriate for the nurse to include in the plan of care?
a. The patient will be free from injury.
b. The patient will receive immunizations.
c. The patient will have adequate oxygenation.
d. The patient will maintain intact perineal skin.

A

ANS: D
The major manifestation of M. avium infection is loose, watery stools, which would increase the risk for perineal skin breakdown. The other outcomes would be appropriate for other complications (e.g., pneumonia, dementia, influenza) associated with HIV infection.

133
Q

Which of these patients who have arrived at the human immunodeficiency virus (HIV) clinic should the nurse assess first?
a. Patient whose rapid HIV-antibody test is positive
b. Patient whose latest CD4+ count has dropped to 250/µL
c. Patient who has had 10 liquid stools in the last 24 hours
d. Patient who has nausea from prescribed antiretroviral drugs

A

ANS: C
The nurse should assess the patient for dehydration and hypovolemia. The other patients also will require assessment and possible interventions, but do not require immediate action to prevent complications such as hypovolemia and shock

134
Q

An older adult with chronic human immunodeficiency virus (HIV) infection who takes medications for coronary artery disease and hypertension has chosen to begin early antiretroviral therapy (ART). Which information will the nurse include in patient teaching?
a. Many drugs interact with antiretroviral medications.
b. HIV infections progress more rapidly in older adults.
c. Less frequent CD4+ level monitoring is needed in older adults.
d. Hospice care is available for patients with terminal HIV infection.

A

ANS: A

135
Q

The registered nurse (RN) caring for an HIV-positive patient admitted with tuberculosis can delegate which action to unlicensed assistive personnel (UAP)?
a. Teach the patient how to dispose of tissues with respiratory secretions.
b. Stock the patient’s room with the necessary personal protective equipment.
c. Interview the patient to obtain the names of family members and close contacts.
d. Tell the patient’s family members the reason for the use of airborne precautions.

A

ANS: B
A patient diagnosed with tuberculosis would be placed on airborne precautions. Because all health care workers are taught about the various types of infection precautions used in the hospital, the UAP can safely stock the room with personal protective equipment. Obtaining contact information and patient teaching are higher-level skills that require RN education and scope of practice

136
Q

The nurse designs a program to decrease the incidence of human immunodeficiency virus (HIV) infection in the adolescent and young adult populations. Which information should the nurse assign as the highest priority?
a. Methods to prevent perinatal HIV transmission
b. Ways to sterilize needles used by injectable drug users
c. Prevention of HIV transmission between sexual partners
d. Means to prevent transmission through blood transfusions

A

ANS: C
Sexual transmission is the most common way that HIV is transmitted. The nurse should also provide teaching about perinatal transmission, needle sterilization, and blood transfusion, but the rate of HIV infection associated with these situations is lower.

137
Q

A nurse is performing the admission assessment of a patient who has AIDS. What components should the nurse include in this comprehensive assessment? Select all that apply.

A) Current medication regimen
B) Identification of patient’s support system
C) Immune system function
D) Genetic risk factors for HIV
E) History of sexual practices

A

Ans: A, B, C, E
Nursing assessment includes numerous focuses, including identification of medication use, support system, immune function and sexual history. HIV does not have a genetic component.

138
Q

A patient who is scheduled for a breast biopsy asks the nurse the difference between a benign tumor and a malignant tumor. Which answer by the nurse is correct?
a. “Benign tumors do not cause damage to other tissues.”
b. “Benign tumors are likely to recur in the same location.”
c. “Malignant tumors may spread to other tissues or organs.”
d. “Malignant cells reproduce more rapidly than normal cells.”

A

ANS: C
The major difference between benign and malignant tumors is that malignant tumors invade adjacent tissues and spread to distant tissues and benign tumors do not metastasize. The other statements are inaccurate. Both types of tumors may cause damage to adjacent tissues. Malignant cells do not reproduce more rapidly than normal cells. Benign tumors do not usually recur.

139
Q

The nurse administers an IV vesicant chemotherapeutic agent to a patient. Which action is most important for the nurse to take?
a. Infuse the medication over a short period of time.
b. Stop the infusion if swelling is observed at the site.
c. Administer the chemotherapy through a small-bore catheter.
d. Hold the medication unless a central venous line is available.

A

ANS: B
Swelling at the site may indicate extravasation, and the IV should be stopped immediately. The medication generally should be given slowly to avoid irritation of the vein. The size of the catheter is not as important as administration of vesicants into a running IV line to allow dilution of the chemotherapy drug. These medications can be given through peripheral lines, although central vascular access devices are preferred.

140
Q

The nurse obtains information about a hospitalized patient who is receiving chemotherapy for colorectal cancer. Which information about the patient alerts the nurse to discuss a possible change in cancer therapy with the health care provider?
a. Frequent loose stools
b. Nausea and vomiting
c. Elevated white blood count (WBC)
d. Increased carcinoembryonic antigen (CEA)

A

ANS: D
An increase in CEA indicates that the chemotherapy is not effective for the patient’s cancer and may need to be modified. Gastrointestinal adverse effects are common with chemotherapy. The nurse may need to address these, but they would not necessarily indicate a need for a change in therapy. An elevated WBC may indicate infection but does not reflect theeffectiveness of the colorectal cancer therap

141
Q

A patient who is being treated for stage IV lung cancer tells the nurse about new-onset back pain. Which action should the nurse take first?
a. Give the patient the prescribed PRN opioid.
b. Assess for sensation and strength in the legs.
c. Notify the health care provider about the symptoms.
d. Teach the patient how to use relaxation to reduce pain.

A

ANS: B
Spinal cord compression, an oncologic emergency, can occur with invasion of tumor into the epidural space. The nurse will need to assess the patient further for symptoms such as decreased leg sensation and strength and then notify the health care provider. Administration of opioids or the use of relaxation may be appropriate but only after the nurse has assessed forpossible spinal cord compression

142
Q

The nurse is caring for a patient with left-sided lung cancer. Which finding would be most important for the nurse to report to the health care provider?
a. Hematocrit of 32%
b. Pain with deep inspiration
c. Serum sodium of 126 mEq/L
d. Decreased breath sounds on left side

A

ANS: C
The syndrome of inappropriate antidiuretic hormone (and the resulting hyponatremia) is an oncologic metabolic emergency and requires rapid treatment to prevent complications such as seizures and coma. The other findings also require intervention but are common in patients with lung cancer and not immediately life threatening

143
Q

An older adult patient who has colorectal cancer is receiving IV fluids at 175 mL/hr in conjunction with the prescribed chemotherapy. Which finding by the nurse is most important to report to the health care provider?
a. Patient complains of severe fatigue.
b. Patient voids every hour during the day.
c. Patient takes only 50% of meals and refuses snacks.
d. Patient has crackles up to the midline posterior chest.

A

ANS: D
Rapid fluid infusions may cause heart failure, especially in older patients. The other findings are common in patients who have cancer or are receiving chemotherapy

144
Q

After change-of-shift report on the oncology unit, which patient should the nurse assess first?
a. Patient who has a platelet count of 82,000/µL after chemotherapy
b. Patient who has xerostomia after receiving head and neck radiation
c. Patient who is neutropenic and has a temperature of 100.5° F (38.1° C)
d. Patient who is worried about getting the prescribed long-acting opioid on time

A

ANS: C
Temperature elevation is an emergency in neutropenic patients because of the risk for rapid progression to severe infections and sepsis. The other patients also require assessments or interventions but do not need to be assessed as urgently. Patients with thrombocytopenia do not have spontaneous bleeding until the platelets are 20,000/µL. Xerostomia does not require immediate intervention. Although breakthrough pain needs to be addressed rapidly, the patientdoes not appear to have breakthrough pain

145
Q

After a patient experienced a brief episode of tinnitus, diplopia, and dysarthria with no residual effects, the nurse anticipates teaching the patient about
a. cerebral aneurysm clipping.
b. heparin intravenous infusion.
c. oral low-dose aspirin therapy.
d. tissue plasminogen activator (tPA).

A

ANS: C
The patient’s symptoms are consistent with transient ischemic attack (TIA), and drugs that inhibit platelet aggregation are prescribed after a TIA to prevent a stroke. Continuous heparin infusion is not routinely used after TIA or with acute ischemic stroke. The patient’s symptoms are not consistent with a cerebral aneurysm. tPA is used only for acute ischemic stroke, not forTIA.

146
Q

A patient is being admitted with a possible stroke. Which information from the assessment indicates that the nurse should consult with the health care provider before giving the prescribed aspirin?
a. The patient has dysphasia.
b. The patient has atrial fibrillation.
c. The patient reports that symptoms began with a severe headache.
d. The patient has a history of brief episodes of right-sided hemiplegia.

A

ANS: C
A sudden onset headache is typical of a subarachnoid hemorrhage, and aspirin is contraindicated. Atrial fibrillation, dysphasia, and transient ischemic attack are not contraindications to aspirin use

147
Q

A patient with a stroke experiences facial drooping on the right side and right-sided arm and leg paralysis. When admitting the patient, which clinical manifestation will the nurse expect tofind?
a. Impulsive behavior
b. Right-sided neglect
c. Hyperactive left-sided tendon reflexes
d. Difficulty comprehending instructions

A

ANS: D
Right-sided paralysis indicates a left-brain stroke, which will lead to difficulty with comprehension and use of language. The left-side reflexes are likely to be intact. Impulsive behavior and neglect are more likely with a right-side stroke

148
Q

During the change of shift report, a nurse is told that a patient has an occluded left posterior cerebral artery. The nurse will anticipate that the patient may have:
a. dysphasia.
b. confusion.
c. visual deficits.
d. poor judgment.

A

ANS: C
Visual disturbances are expected with posterior cerebral artery occlusion. Aphasia occurs withmiddle cerebral artery involvement. Cognitive deficits and changes in judgment are more typical of anterior cerebral artery occlusion.

149
Q

When teaching about clopidogrel (Plavix), the nurse will tell the patient with cerebral atherosclerosis
a. to monitor and record the blood pressure daily.
b. to call the health care provider if stools are tarry.
c. that clopidogrel will dissolve clots in the cerebral arteries.
d. that clopidogrel will reduce cerebral artery plaque formation.

A

ANS: B
Clopidogrel inhibits platelet function and increases the risk for gastrointestinal bleeding, so patients should be advised to notify the health care provider about any signs of bleeding. The medication does not lower blood pressure, decrease plaque formation, or dissolve clots

150
Q

For a patient who had a right hemisphere stroke, the nurse anticipates planning interventions to manage
a. impaired physical mobility related to right-sided hemiplegia.
b. risk for injury related to denial of deficits and impulsiveness.
c. impaired verbal communication related to speech-language deficits.
d. ineffective coping related to depression and distress about disability.

A

ANS: B
The patient with right-sided brain damage typically denies any deficits and has poor impulse control, leading to risk for injury when the patient attempts activities such as transferring froma bed to a chair. Right-sided brain damage causes left hemiplegia. Left-sided brain damage typically causes language deficits. Left-sided brain damage is associated with depression and distress about the disability.

151
Q

When caring for a patient with a new right-sided homonymous hemianopsia resulting from a stroke, which intervention should the nurse include in the plan of care?
a. Apply an eye patch to the right eye.
b. Approach the patient from the right side.
c. Place needed objects on the patient’s left side.
d. Teach the patient that the left visual deficit will resolve.

A

ANS: C
During the acute period, the nurse should place objects on the patient’s unaffected side. Because there is a visual defect in the right half of each eye, an eye patch is not appropriate. The patient should be approached from the left side. The visual deficit may not resolve, although the patient can learn to compensate for the defect

152
Q

A patient will attempt oral feedings for the first time after having a stroke. The nurse should assess the gag reflex and then
a. order a varied pureed diet.
b. assess the patient’s appetite.
c. assist the patient into a chair.
d. offer the patient a sip of juice.

A

ANS: C
The patient should be as upright as possible before attempting feeding to make swallowing easier and decrease aspiration risk. To assess swallowing ability, the nurse should initially offer water or ice to the patient. Pureed diets are not recommended because the texture is too smooth. The patient may have a poor appetite, but the oral feeding should be attempted

153
Q

Which stroke risk factor for a 48-yr-old male patient in the clinic is most important for the nurse to address?
a. The patient is 25 lb above the ideal weight.
b. The patient drinks a glass of red wine with dinner daily.
c. The patient’s usual blood pressure (BP) is 170/94 mm Hg.
d. The patient works at a desk and relaxes by watching television.

A

ANS: C
Hypertension is the single most important modifiable risk factor. People who drink more than 1 (for women) or 2 (for men) alcoholic beverages a day may increase their risk for hypertension. Physical inactivity and obesity contribute to stroke risk but not as much as hypertension

154
Q

A patient with left-sided weakness that started 60 minutes earlier is admitted to the emergencydepartment and diagnostic tests are ordered. Which test should be done first?
a. Complete blood count (CBC)
b. Chest radiograph (chest x-ray)
c. Computed tomography (CT) scan
d. 12-Lead electrocardiogram (ECG)

A

ANS: CRapid screening with a noncontrast CT scan is needed before administration of tissue plasminogen activator (tPA), which must be given within 4.5 hours of the onset of clinical manifestations of the stroke. The sooner the tPA is given, the less brain injury. The other diagnostic tests give information about possible causes of the stroke and do not need to be completed as urgently as the CT scan

155
Q

A patient in the emergency department with sudden-onset right-sided weakness is diagnosed with an intracerebral hemorrhage. Which information about the patient is most important to communicate to the health care provider?
a. The patient’s speech is difficult to understand.
b. The patient’s blood pressure (BP) is 144/90 mm Hg.
c. The patient takes a diuretic because of a history of hypertension.
d. The patient has atrial fibrillation and takes warfarin (Coumadin).

A

ANS: D
The use of warfarin probably contributed to the intracerebral bleeding and remains a risk factor for further bleeding. Administration of vitamin K is needed to reverse the effects of the warfarin, especially if the patient is to have surgery to correct the bleeding. The history of hypertension is a risk factor for the patient but has no immediate effect on the patient’s care. The BP of 144/90 indicates the need for ongoing monitoring but not for any immediate changein therapy. Slurred speech is consistent with a left-sided stroke, and no change in therapy is indicated

156
Q

Nurses in change-of-shift report are discussing the care of a patient with a stroke who has progressively increasing weakness and decreasing level of consciousness. Which patient problem do they determine has the highest priority for the patient?
a. Impaired physical mobility related to weakness
b. Disturbed sensory perception related to brain injury
c. Risk for impaired skin integrity related to immobility
d. Risk for aspiration related to inability to protect airway

A

ANS: D
Protection of the airway is the priority of nursing care for a patient having an acute stroke. Theother diagnoses are also appropriate, but interventions to prevent aspiration are the priority at this time

157
Q

Which information about the patient who has had a subarachnoid hemorrhage is most important to communicate to the health care provider?
a. The patient complains of having a stiff neck.
b. The patient’s blood pressure (BP) is 90/50 mm Hg.
c. The patient reports a severe and unrelenting headache.
d. The cerebrospinal fluid (CSF) report shows red blood cells (RBCs).

A

ANS: B
To prevent cerebral vasospasm and maintain cerebral perfusion, BP needs to be maintained at a level higher than 90 mm Hg systolic after a subarachnoid hemorrhage. A low BP or drop in BP indicates a need to administer fluids and/or vasopressors to increase the BP. An ongoing headache, RBCs in the CSF, and a stiff neck are all typical clinical manifestations of a subarachnoid hemorrhage and do not need to be rapidly communicated to the health care provider

158
Q

A 70-yr-old female patient with left-sided hemiparesis arrives by ambulance to the emergency department. Which action should the nurse take first?
a. Take the patient’s blood pressure.
b. Check the respiratory rate and effort.
c. Assess the Glasgow Coma Scale score.
d. Send the patient for a computed tomography (CT) scan.

A

ANS: B
The initial nursing action should be to assess the airway and take any needed actions to ensurea patent airway. The other activities should take place quickly after the ABCs (airway, breathing, and circulation) are completed

159
Q

A 63-yr-old patient who began experiencing right arm and leg weakness is admitted to the emergency department. In which order will the nurse implement these actions included in the stroke protocol? (Put a comma and a space between each answer choice [A, B, C, D].)
a. Obtain CT scan without contrast.
b. Infuse tissue plasminogen activator (tPA).
c. Administer oxygen to keep O2 saturation >95%.
d. Use National Institute of Health Stroke Scale to assess patient.

A

ANS: C, D, A, B
The initial actions should be those that help with airway, breathing, and circulation. Baseline neurologic assessments should be done next. A CT scan will be needed to rule out hemorrhagic stroke before tPA can be administered

160
Q

he nurse notes a reddened area on the right heel that does not turn lighter in color when pressed with a finger. Which term will the nurse use to describe this area?
a. Reactive hyperemia
b. Secondary erythema
c. Blanchable hyperemia
d. Nonblanchable erythema

A

ANS: D
Nonblanchable erythema is redness that persists after palpation and indicates tissue damage. When you press a finger against the red or purple area, it does not turn lighter in color. Deep tissue damage is present and is commonly the first stage of pressure injury development. There is no such condition as secondary erythema. Reactive hyperemia is a redness of the skin resulting from dilation of the superficial capillaries. Reactive hyperemia blanches. In blanchable hyperemia, the area that appears red and warm will blanch (turn lighter in color) following fingertip palpation

161
Q

Which assessment finding indicates to the nurse that the patient is at high risk for developing a pressure injury?
a. Serum total protein level of 4.6 g/dL
b. Braden Scale score of 22
c. Cetirizine 5 mg PO daily
d. Fasting serum glucose level 84 mg/dL

A

ANS: A Poor nutrition, specifically severe protein deficiency, causes soft tissue to become susceptible to breakdown. Serum protein level of 4.6 g/dL leads to edema or swelling, which contributes to problems with the transportation of oxygen and nutrients. A hospitalized adult with a score of 16 or below and an older adult at 18 or below are at risk for pressure injury development; a score of 22 does not place the patient at risk. Cetirizine is a nonsedating antihistamine medication that would not place the patient at risk for developing pressure injuries. Fasting serum glucose level of 84 mg/dL is within normal limits and does not indicate risk of pressure injury development.

162
Q

The patient’s sacral pressure injury is open with exposed bone. Which pressure injury stage will be recorded in the patient’s chart?
a. Stage 1
b. Stage 2
c. Stage 3
d. Stage 4

A

ANS: D
Stage 4: Full-thickness tissue loss with exposed bone, tendon, or muscle; slough or eschar may be present; often includes undermining and tunneling
Stage 1: Intact skin with nonblanchable redness of a localized area, usually over a bony prominence
Stage 2: Partial-thickness loss of dermis presenting as a shallow open ulcer with a red-pink wound bed, without slough; may also present as an intact or open/ruptured serum-filled blister
Stage 3: Full-thickness tissue loss; subcutaneous fat may be visible, but bone, tendon, or muscle is not exposed; slough may be present but does not obscure the depth of tissue loss; may include undermining and tunneling

163
Q

The patient has a large red, blistered area on the left hip. Which pressure injury stage will be recorded in the patient’s chart?
a. Stage 1
b. Stage 2
c. Stage 3
d. Stage 4

A

ANS: B
Stage 2: Partial-thickness loss of dermis presenting as a shallow open ulcer with a red-pink wound bed, without slough; may also present as an intact or open/ruptured serum-filled blister

164
Q

The patient has a large left hip decubitus ulcer with tunneling but no involvement of bone, tendon, or muscle. Which pressure injury stage will be recorded in the patient’s chart?
a. Stage 1
b. Stage 2
c. Stage 3
d. Stage 4

A

ANS: C
Stage 3: Full-thickness tissue loss; subcutaneous fat may be visible, but bone, tendon, or muscle is not exposed; slough may be present but does not obscure the depth of tissue loss; may include undermining and tunneling

165
Q

The nurse is caring for a patient who has perineal skin breakdown after sitting in wet underclothes for many hours. Which term will be used to document the patient’s condition in the medical record?
a. Maceration
b. Dehiscence
c. Evisceration
d. Debridement

A

ANS: A
Maceration is skin breakdown due to extended exposure to moisture. Dehiscence is the partial or total separation of layers of skin and tissue above the fascia in a wound that is not healing properly. Evisceration occurs when wound layers separate below the fascial layer, and visceral organs protrude through the wound opening. Debridement is removal of dead or infected tissue from a wound

166
Q

Which is the first intervention of the nurse for changing the dressing to a painful burn?
a. Administer pain medication 30 minutes beforehand.
b. Gently irrigate the wound using sterile normal saline.
c. Loosen the tape gently by pressing the skin away from it.
d. Observe the wound bed for presence of granulation tissue.

A

ANS:
A When you plan a dressing change, consider giving the patient an analgesic at least 30 minutes beforehand. The tape should be loosened gently by pressing the skin away from it. The wound bed should be observed and then irrigation should be performed.

167
Q

The patient’s incision is fading to a pale pink following surgery 2 months previously. Which stage of the healing describes the current status of the patient’s wound?
a. Hemostasis phase
b. Remodeling phase
c. Proliferative phase
d. Inflammation phase

A

ANS: B
The patient’s wound is at the remodeling phase as the healing process continues to strengthen the scar tissue in the wound. The proliferative phase causes new capillary networks to form that provide oxygen and nutrients for new tissue and contribute to the synthesis of collagen. In the inflammation phase the goal is to establish a clean wound bed and obtain bacterial balance. Hemostasis phase controls bleeding.

168
Q

Which is the priority nursing assessment for a patient wearing an abdominal binder after abdominal surgery?
a. Mental status and orientation
b. Hourly fluid intake and output
c. Lung sounds and pulse oximetry
d. Presence of peripheral pedal pulses

A

ANS: C
Evaluate the patient’s ability to ventilate properly, including deep breathing and coughing. An abdominal binder supports a large incision that is vulnerable to stress when a patient moves or coughs. Mental status, fluid balance, and peripheral pulses are not affected by the abdominal binder

169
Q

The patient’s wound has thick creamy yellow drainage present on the dressing. How will the nurse document this finding?
a. Serous drainage
b. Purulent drainage
c. Sanguineous drainage
d. Serosanguineous drainage

A

ANS: B
Purulent is thick, yellow, green, or brown, indicating the presence of dead or living organisms and white blood cells. Serosanguineous is pale, more watery, and a combination of plasma and red cells, which may be blood streaked. Serous is clear, watery plasma. Sanguineous is fresh bleeding

170
Q

The patient just sustained a deep laceration that is bleeding profusely. Which stage of healing describes the current state of the patient’s wound?
a. Hemostasis phase
b. Proliferative phase
c. Inflammation phase
d. Remodeling phase

A

ANS: A
Hemostasis phase controls bleeding. Inflammation phase establishes a clean wound bed and obtain bacterial balance. Proliferative phase produces new tissue, epithelialization, and contraction. Remodeling phase reorganizes the collagen to produce a more elastic, stronger collagen for the scar tissue.

171
Q

On admission a patient is noted to have an alteration in skin integrity on the right heel. The nurse uses the Braden Scale. Which areas will the nurse assess when using this scale? (Select all that apply.)
a. Mobility
b. Nutrition
c. Infection
d. Activity
e. Friction

A

ANS: A, B, D, E
The Braden Scale is a highly reliable scale that uses six subscales to identify patients at greatest risk for pressure injuries: sensory perception, moisture, activity, mobility, nutrition, and friction and shear. Infection is not an area that is assessed on the Braden Scale

172
Q

Which outcomes are appropriate for the patient with the nursing diagnosis risk for impaired skin integrity related to immobility and muscle weakness? (Select all that apply.)
a. The patient’s skin will remain intact without redness or ulceration.
b. The nurse will assess the patient’s skin daily for any sign of breakdown.
c. The patient will verbalize at least two methods to prevent skin breakdown.
d. The patient’s wounds will be kept clean and will not develop signs of infection. e. The nurse will reposition the patient every 2 hours and pad bony prominences.

A

ANS: A, C
Appropriate goals for risk for impaired skin integrity include maintenance of intact skin and patient verbalization of methods to prevent skin breakdown. Daily wound assessment and repositioning are interventions. The presence of wounds makes the risk for impaired skin integrity nursing diagnosis inappropriate as wounds have already developed

173
Q

A postoperative abdominal surgery patient has been admitted to the surgical floor. The nurse is aware that wound healing is delayed due to complications. Which conditions would prevent normal wound healing at the surgical site? (Select all that apply.)
a. Dehiscence
b. Evisceration
c. Debridement
d. Hemostasis
e. Hemorrhage

A

ANS: A, B, E
Complications of wound healing include any of the following: hemorrhage, hematoma, infection, dehiscence, and evisceration. Hemostasis is a normal response to healing, not a complication. Debridement is the removal of dead or infected tissue from the wound which will promote healing

174
Q

Give two reasons for inserting an IV line.
(Case study #13, Angina)

A

To have IV access in case of cardiac arrest, for bolus fluid administration, and for administration of IV nitroglycerin (NTG) or other medications as needed

175
Q

Explain the purpose of the aspirin tablet. Why is “non–enteric-coated” aspirin specified? What would be a contraindication to administering aspirin?
(Case study #13, Angina)

A

Aspirin is effective in reducing mortality in MI due to its antiplatelet action

An enteric coating slows down the availability of aspirin, and in this case, it is desirable for the medication to be rapidly absorbed for rapid availability in the bloodstream. Chewing the tablet also enhances the absorption.

Contraindications:
allergy to aspirin
active major bleeding or major bleeding within the past 2 weeks,
hemorrhagic stroke
active or recent gastric bleeding, ulcer disease,
acute bronchospasm

176
Q

While you care for R.K., you carefully observe her. Identify two possible coronary artery disease (CAD) complications and the associated signs and symptoms.
(Case study #13, Angina)

A

Cardiac ischemia: chest pain or discomfort, often described as squeezing, tight, heavy or suffocating. Pain radiating to other areas (shoulder, jaw, arm etc)

Cardiac dysrhythmias, particularly premature ventricular contractions: dysrhythmias seen on telemetry, syncope, lightheadedness, shortness of breath, palpitations felt by patient

HF: crackles; dyspnea; confusion; dry, hacking, nonproductive cough; peripheral edema; JVD

177
Q

What essential safety point will you emphasize when discussing sublingual nitro?
(Case study #13, Angina)

A

It is essential to sit or lie down when taking sublingual nitroglycerin because it will cause a drop in BP, she will be at a higher risk for falling

Should be dissolved under tongue not swallowed; do not take any oral meds when sublingual nitro in place

178
Q

Evaluate the Arterial Blood Gases (room air). What is your interpretation of the ABG’s and what do you think the Dr. will order next and why? (Case study #47, Post-Op PE)

pH 7.55
PaC02 24 mm Hg
HCO3 24 mEq/L
Pao2 56 mm Hg
Sao2 86% (room air)

A

pH 7.55 – high = alkalosis
PaC02 24 mm Hg – low = opposite to PH therefore is a primary respiratory disorder (in metabolic pH and PaCO2 move in the same direction)
HCO3 24 mEq/L – normal
Pao2 56 mm Hg – low = lungs unable to oxygenate blood causing low oxygen tension in arterial blood
Sao2 86% (room air) – low – hypoxia

-

The ABGs show an acute respiratory alkalosis with hypoxemia.
The physician will probably order a spiral (or helical) CT scan to evaluate for a pulmonary embolus (PE).
Evaluation for DVT may be performed.
D-dimer blood test may also be ordered but other conditions may cause elevation; many patients with small pulmonary emboli have normal D-Dimer results

179
Q

The physician is considering administering an antidote to the heparin. Which generic drug is considered an antidote to heparin therapy?
a. atropine
b. Vitamin K
c. Protamine sulfate
d. potassium chloride
(Case study #47, Post-Op PE)

A

c. Protamine sulfate is the antidote to heparin overdose; (Vit K is the antidote to warfarin overdose).

180
Q

What common side effects of bronchodilator therapy do you need to assess for? (Case study #33, COPD)

A

Tremors
Tachycardia
Anxiety
Nausea
Palpitations

181
Q

Describe albuterol (class and mechanism of action/indication).
(Case study #33, COPD)

A

Albuterol is a fast-acting beta2 agonist that acts to relax and open airways and increase ciliary movement to help clear secretions.

182
Q

Describe ipratropium (class and mechanism of action/indication).
(Case study #33, COPD)

A

Ipratropium is an anticholinergic that causes bronchodilation and inhibits secretions without causing systemic anticholinergic effects. The combination is more effective than either drug alone.

183
Q

Describe Fluticasone/salmeterol (class and mechanism of action/indication).
(Case study #33, COPD)

A

Fluticasone/salmeterol (Advair) 100/50 is a combination drug containing a low-dose inhaled steroid (fluticasone 100 mcg) that decreases the inflammation of the airways and a long-acting beta agonist (salmeterol 50 mcg) that acts as a bronchodilator. This pairing is useful in reducing swelling, mucus production, and spasm in and of the airways resulting in the easing of airway constriction.

184
Q

Interpret these ABG values:
pH - 7.33
PaCO2 - 58 mm Hg
HCO3 - 32 mEq/L
PaO2 - 65 mm Hg
SaO2 - 92%

(Case study #33, COPD)

A

pH level - normal/slightly below
PaCO2 level - elevated
bicarbonate (HCO3) level - high
PaO2 & SAT - low

Interpretation:
Partially compensation respiratory acidosis
Kidneys have compensated for high CO2 levels by increasing HCO3 levels to attain a near normal PH level. If he had been developing respiratory failure, the ph would be acidotic and PAO2 would have decreased, resulting in uncompensated respiratory acidosis and tissue hypoxia.

185
Q

What is Sickle Cell Disease? (Case study #27, SCD)

A

SCD is a group of inherited, autosomal recessive disorders characterized by the presence of an abnormal form of hemoglobin (HgB S) in RBCs. The abnormal Hgb causes the RBCs to elongate or stiffen into a sick shape when O2 levels are low. Repeated deoxygenation results in permanent sickling of RBCs. Vascular inflammation and abnormal adhesion also increase vasoocclusion.

186
Q

True or False?
1. Only Blacks get SCD.
2. There is currently no cure for SCD.
3. Those with SCD should not receive childhood vaccinations.
4. SCD cannot be diagnosed before an infant is 3 to 4 months old.
5. An ophthalmologist should perform an eye examination every 1 to 2 years.
6. Those with SCD should fly only on airplanes with pressurized cabins.

(Case study #27, SCD)

A

1, 2, 3, 4 are false
5 and 6 are true.

187
Q

Which statement is true about the inheritance pattern of SCD?
1. If ’s K.J. mother has sickle cell trait and her father has SCD, each child will either have SCD or have sickle cell trait/be a carrier.
2. If K.J.’s mother has sickle cell trait and her father is normal, each child will have sickle cell trait.
3. If both K.J.’s parents have SCD, each child will have with either have SCD or be normal.
4. If K.J. has children, each child will automatically have SCD regardless of her spouses status.

(Case study #27, SCD)

A

ANS: A
If her mother has sickle cell trait and her father has SCD, child’s risk is 50% for either having SCD or being a carrier

SCD is an autosomal recessive disease

188
Q

Why is anemia common in patients with SCD?
(Case study #27, SCD)

A

The lifespan of RBCs is significantly reduced in SCD from 120 days to about 10-20 days, so the bone marrow compensates with increased production.

Reticulocytes (immature RBCs) gives an indication of RBC production by the bone marrow.

Sickled RBCs are more friable and rupture more easily than normal RBCs.

189
Q

What is the importance of CD4-cells and HIV viral load counts?
(Case study #104, HIV/AIDS)

A

CD4 cells:
white blood cells calledT lymphocytesor T cells that fight infection and play an important role inimmune systemfunction.CD4 tests measure the number of these cells in the blood and, in conjunction with anHIV viral load test, help assess the status of the immune system in a person who has been diagnosed with HIV. CD4 counts also identify the risk for opportunistic infection.

HIV Viral load:
determines the amount of HIV virus in the blood of the infected person
Accurate marker for prognosis, disease progression, response to treatment, and indication for antiretroviral prophylactic treatment.

190
Q

What type of precautions do you need to implement when caring for a patient with an HIV/AIDS diagnosis?
(Case study #104, HIV/AIDS)

A

Standard precautions for blood and bodily fluids are all that is necessary

191
Q

Why might an patient with HIV be placed on Bactrim? What major antibiotic side effects do you need to monitor for?
(Case study #104, HIV/AIDS)

A

Used to treat PJP (opportunistic fungal infection) and prevent recurrence

50% of AIDS patients experience the following adverse reactions when taking Bactrim:
vomiting,
hyponatremia,
fever, rash,
neutropenia, thrombocytopenia,
hepatitis
jaundice.

192
Q

List 3 opportunistic infections.
(Case study #104, HIV/AIDS)

A
  • Candidiasis, fungal infection. Commonly proceeds other life threatening infections
  • Cryptosporidiosis, parasitic organism associated with HIV, diarrhea
  • Cryptococcal meningitis
  • Toxoplasmosis, parasitic flu like symptoms
  • Mycoplasma pneumonia from mycobacterium
  • CMV
  • Cytomegalovirus Retinitis; rx meds may cause severe neutropenia
  • Herpes simplex, viral
  • Histoplasmosis- fungus; fever, chills, cough rash
  • Tuberculosis (TB) Mycobacterium; (now have increasing drug resistant TB)
  • Hepatitis
193
Q

Describe 5 common manifestations of a stroke.
(Case study #78, CVA)

A

facial dropping
slurred speach
aphasia
headache
difficultly swallowing
ataxia
double vision
altered LOC
confusion
hypertension

194
Q

Describe the FAST stroke assessment.
(Case study #78, CVA)

A

F = FACE
Ask person to smile. Look for one-sided facial droop.
A = ARMS
Ask person to raise their arms. Look to see if one arm drifts downward.
S = SPEECH
Ask person to repeat a simple phrase. Listen for word slurring and correct repeating of the phrase.
T = TIME
If any deficiencies are noted – facial droop, arm drift, speech slurring – time is critical.

195
Q

What is the primary diagnostic test used for identifying a suspected stroke?
(Case study #78, CVA)

A

Non-contrast computed tomography (CT) scan is the primary test used to diagnose a stroke.

CT can indicate the size and location of the lesion and differentiate between ischemic and hemorrhagic stroke.
For optimal results, the CT scan should be obtained within 25 minutes and read within 45 minutes of arrival at the emergency department. If the stroke is ischemic and is less than 3 hours old, the CT will appear normal because the brain structure with or without blood flow appears the same in a non-contrast CT scan.

196
Q

Why is knowing the type of stroke an important factor in planning care?
(Case study #78, CVA)

A

The primary difference between the types of strokes is in the medical management.

An ischemic stroke, caused by the occlusion of a cerebral artery by either a thrombus or an embolus, can be treated by anticoagulants and/or fibrinolytic therapy. Anticoagulants and fibrinolytic therapy are contraindicated in a patient who has had a hemorrhagic stroke.

197
Q

Which interventions can you delegate to the assistive personnel (AP)? Select all that apply. *equivalent to PSW

  1. Obtaining N.T.’s weight
  2. Obtaining a manual BP per protocol
  3. Initiating O2therapy by nasal cannula
  4. Assisting N.T. in repositioning every 2 hours
  5. Performing N.T.’s neurologic checks every hour

(Case study #78, CVA)

A

ANS: 1, 2, 4
The AP cannot perform a neurologic check or set up oxygen therapy.
All other activities are within the usual scope of practice for an AP in the United States.

198
Q

What signs and symptoms would alert you to the possible presence of an intracranial hemorrhage during tPA infusion?
(Case study #78, CVA)

A

Changes/deterioration in neuro vitals
Sudden increase in blood pressure
Vomiting
New onset of headache
*All are indicative of increase in intracranial pressure

199
Q

What assessment finding would indicate that the body is attempting to increase cerebral blood flow?
1. Increasing BP from baseline
2. Pupils become nonreactive
3. New onset of dysrhythmias
4. Presence of S3and S4heart sounds

(Case study #78, CVA)

A

ANS: A
Increasing BP from baseline – this is an expected complication during tPA infusion which is why guidelines will incl

200
Q

Why would a patient be placed on clopidogrel (Plavix) post-CVA?
(Case study #78, CVA)

A

Clopidogrel plus aspirin is currently recommended to prevent platelet aggregation and thereby prevent CVA and secondary CVA.

201
Q

What are the common sites of lung cancer metastasis?
(Case study #112, Cancer)

A

Bones
Liver
Brain
Lymph nodes
Adrenal glands

202
Q

Using simple terms, how would you explain combination chemotherapy and how it works to a patient?
(Case study #112, Cancer)

A

Chemotherapy works by stopping cancer cells from multiplying. Cells go through several different phases when they multiply. Different chemotherapeutic drugs attack the cancer cells during different phases. Use of chemotherapeutic agents that work during different phases increases the effectiveness of the therapy

203
Q

What are 2 goals of administering chemotherapy to patients who have untreatable/end-stage/terminal cancers?
(Case study #112, Cancer)

A

The first is to prolong survival through controlling or slowing the progression of his cancer.

The second goal is palliation. With palliation, relief or control of symptoms and the maintenance of a satisfactory quality of life are the focus.

204
Q

Describe 4 interventions you would implement to manage dyspnea in a lung cancer patient.

A

Semi/high Fowlers position to allow for maximal lung expansion

Teach him how to use diaphragmatic and pursed-lip breathing

Decrease activity level – teach him to pace himself with activities

Assess anxiety component - consider a referral to psychosocial support services

Utilize Oxygen therapy as needed

Administer medications such as morphine

Encourage maximal fluid intake to maintain hydration

205
Q

Normal lab value/range: Potassium

A

3.5–5.0 mmol/L

206
Q

Normal lab value/range: Sodium

A

135-145 mEq/L

207
Q

Normal lab value/range: WBC count

A

3.5–10.5 × 109/L

208
Q

Normal lab value/range: Hemoglobin (Hgb)

A

Female 115–155 g/L
Male 125–170 g/L

209
Q

Normal lab value/range: Digoxin

A

0.5 to 2 ng/mL

(The toxic level is >2.4 ng/mL)

210
Q

Describe furosemide (class and mechanism of action/indication).

A

Indicated for the treatment of edema associated with congestive heart failure, cirrhosis of the liver, and renal disease, including the nephrotic syndrome. Furosemide may also be used for the treatment of high blood pressure.
This medication may cause dehydration and electrolyte imbalance.

211
Q

Describe Metaprolol (class and mechanism of action/indication).

A

Brand name: Lopressor
Class: beta-blocker
Used alone or in combination with other medications to treat high blood pressure, chronic angina, and improve survival after a heart attack.

212
Q

Describe Digoxin (class and mechanism of action/indication).

A

Class: cardiac glycoside
Indicated to treat and manage heart failure, atrial fibrillation, and atrial flutter.
Digoxin improve the strength and efficiency of the heart, or to control the rate and rhythm of the heartbeat.

213
Q

Describe heparin (class and mechanism of action/indication).

A

Class: anticoagulant
Decrease the clotting ability of the blood - prevention and treatment of thrombotic events such as deep vein thrombosis (DVT), pulmonary embolism (PE), myocardial infarction (MI) as well as atrial fibrillation (AF).

214
Q

Describe atorvastatin (class and mechanism of action/indication).

A

Brand name: Lipitor
HMG-CoA reductase inhibitors, or statins. It works by blocking an enzyme that is needed by the body to make cholesterol, and this reduces the amount of cholesterol in the blood.

215
Q

Describe clopidogrel (class and mechanism of action/indication).

A

Antiplatelet
Reduces the ability of the platelets to stick together and reduces the risk of clots forming. CVA/MI prophylaxis